Lsat Lsat Success
Lsat Lsat Success
Lsat Lsat Success
Thomas O. White
05
04 03
Contents
Red Alert. . . . . . . . . . . . . . . . . . . . . . . . . . . . . . . . . . . . . . . . . . . . . .
Diagnostic LSAT. . . . . . . . . . . . . . . . . . . . . . . . . . . . . . . . . . . . . . . .
Diagnostic Test. . . . . . . . . . . . . . . . . . . . . . . . . . . . . . . . . . . . . . . . . . . . .
Quick-Score Answers. . . . . . . . . . . . . . . . . . . . . . . . . . . . . . . . . . . .
Explanatory Answers . . . . . . . . . . . . . . . . . . . . . . . . . . . . . . . . . . .
13
39
40
51
57
67
77
83
89
99
111
119
129
133
137
141
147
155
CONTENTS
163
191
192
201
229
230
www.petersons.com
iv
LSAT Success
R E D A LERT
ABOUT THE LAW SCHOOL
ADMISSION TEST
RED
CRITICAL FACTORS
PERFORMANCE
IN
TEST
Anxiety
Anxiety results from the mystique of the LSAT and,
for many, past experience with standardized tests.
By confronting the factors that produce anxiety
ALERT
RED ALERT
TRAINING IN THE FUNDAMENTALS IS
CRUCIAL TO LSAT PERFORMANCE.
HIGH-IMPACT TRAINING
PLANNING
AND
Academic Conditioning
Your past academic experience has taught you
that a superior test performance results from
attending lectures, taking and reviewing notes,
reading, cramming, and memorizing everything
you can. Naturally you conclude that the path to
a superior LSAT score is the samelistening to
lectures, taking and reviewing notes, reading,
memorizing, and cramming into your head
everything you can about the LSAT.
Wrong! The LSAT is not an academic
exercise and cannot be converted into one.
Working to convert the LSAT into something
familiar is a waste of time. Getting ready for the
LSAT is much more like athletics, business, or
the military, in all of which a superior performance results from superior training and
planning, not learning. The training techniques
you will learn in this book will permit you to
sidestep the negative consequences of your
academic training.
A FOUNDATION OF LSAT-SPECIFIC
TECHNIQUES PRODUCES THE HIGHEST
IMPACT ON YOUR SCORE.
When you acquire a new skill, the gains are
often great. For example, after training in
computer basics for a short time, you find you
can get results you never could before. So it is
with training in LSAT basics: If you use the
techniques you will learn here to train actively
for a short time, you will find you get big results.
Then, and only then, will you be ready for
practice with test questions and other theories of
preparation.
Reasoning
The LSAT tests your skill at being able to think
like a lawyerreasoning in the principal ways
law schools and the legal system require. This
way of thinking is generally unfamiliar to people
who are considering a legal education, and the
LSAT attempts to measure their aptitude for it.
By providing you with techniques designed to
respond to the reasoning requirements of the
LSAT, this book will enable you to perform more
successfully on every aspect of the test.
www.petersons.com
RED
ALERT
LSAT Success
RED ALERT
Day Assignment
Day Assignment
1
2
3
4
5
6
7
8
9
10
11
12
13
14
15
16
17
18
19
20
21
22
23
24
25
26
27
28
29
30
31
32
33
34
35
RA 1
Positioning
Diagnostic
RA 2
S1
S2
S3
RA 3
S4
S5
S6
S7
S8
S9
S 10
QS Relationships
QS Relationships
QS Relationships
QS Relationships
S 11
QS Arguments
QS Arguments
QS Arguments
QS Arguments
S 12
QS Passages
QS Passages
QS Passages
QS Passages
S 13
S 14
WS 1
WS 2
S 15
Simulated LSAT
LSAT Success
RED
Day Assignment
13
14
15
16
17
18
19
20
21
22
23
24
RA 1
Positioning
RA 2
S1
S 2, S 3
RA 3, S 4
S5
S6
S7
S8
S9
S 10
ALERT
www.petersons.com
RED ALERT
Day Assignment
1 RA 1,
Positioning
2 RA 2, S 1
3 S 2, S 3
4 RA 3, S 4
5 S 5, S 6
6 S7
7 S8
8 S9
9 S 10
Day Assignment
1
Day
Assignment
Day
RA 1,
Diagnostic
RA 2, S 1
8
9
S 2, S 3
10
RA 3, S 4
11
S 5, S 6
12
S 7, S 8, S 9
S 10, QS4
Relationships sets
www.petersons.com
RA 2, S 1, S 2, S 3,
RA 3, S 4
S 5, S 6, S 7,
S 8, S 9
S 10, QS4
Relationships sets
3
S 11
QS4 Arguments sets
S 12
QS4 Passages sets
S 13
S 14, WS 1
S 15
Simulated LSAT
S 11, QS4
Arguments sets
S 12, QS4
Passages sets
S 13, S 14
WS 1, S 15
Simulated LSAT
Assignment
Success
RA 2
S1
S3
RA 3
S4
S5
Assignment
RED
S 11, QS4
Arguments sets
S 12, QS4
Passages sets
S 13, S 14, S 15
3
4
5
6
7
RA 1, Diagnostic
Day Assignment
11
12
13
14
15
16
17
18
Day Assignment
S
S
S
S
S
S
S
6
7
8
9
13
14
15
GETTING STARTED
First, complete the Diagnostic LSAT to get a feel
for the types of questions you will encounter and
an assessment of your strengths and weaknesses.
The next success sessions deal with two critical
factors in test performance that do not appear
directly on the LSAT: anxiety and academic
conditioning. After this, the sessions concentrate
on elements of the LSAT and the type of
reasoning required by law schools and the legal
system. The final sessions discuss techniques for
handling the LSAT Writing Sample and the test
environment.
ALERT
LSAT Success
RED ALERT
There are two LSAT practice tests included
in this book, as well as two more on the CD,
each of which contains five complete sections of
LSAT questions. You should train with practice
questions. A limited number of practice ques-
LSAT Success
RED
ALERT
www.petersons.com
Diagnostic LSAT
BEFORE YOU BEGIN: ESTABLISHING YOUR SUCCESS POSITION
work area. No food, drink, smokes, or similar
digressions are permitted. Pencils with erasers
and a watch, clock, or other precise timing
device are essential. No reference materials,
scratch paper, or outside help may be used.
To reiterate, the LSAT score is determined
only by the number of best answers marked on
the answer sheet. Unlike similar standardized
tests, there is no penalty for guessing when you
dont know the answer. Omitted answers count
as wrong answers. Selecting a wrong answer is
no worse than omitting an answer, which means
that you should never omit an answer, whether
on the actual LSAT or in these introductory
exercises.
WHAT
EXERCISES?
The test-maker is interested in results only.
Simply put, this means that you get credit for
only the number of best answers the scoring
machine finds on your answer sheet.
WHAT IS
SHEET?
HOW
HOW
ANSWER SHEET?
POSITIONING EXERCISES?
Before beginning, make certain that you will not
be interrupted during the time required to
complete the test or test sections. Turn off the
phone, hang out your Do Not Disturb or Privacy
Please sign, and secure the perimeter of your
DIAGNOSTIC LSAT
plete the Answer and Analysis Sheet by following
the instructions below. Once completed, you will
analyze the data on the form and learn more
about where you stand with respect to the LSAT.
On the Answer and Analysis Sheet, you will
find a series of rows and columns. Record the
data required down each column as you work
your way across the rows. Place the answer
choices you made in each exercise in the spaces
provided.
A B C D E G T
j jjj j j j
Situation 1
Situation 1 involves the certain answer selection
that you make when you are sure of the best
answer to a question. You blacken the space on
the answer sheet that corresponds to your
answer selection for that question.
Situation 2
Situation 2 involves guessing after consideration
of the question and the answer options. When
there are more than two answer options about
which you are uncertain, make an additional
mark on your answer sheet.
For example, suppose you consider
question 1 and determine that answer option B is
wrong, but you cannot narrow the other options.
In fact, you cannot decide among the four
remaining optionsA, C, D, and E.
Recording Guesses
Recording guessing results requires you to
review both your original answers and the Best
Match column. If you Xd a G on your answer
sheet, place a Y in the corresponding space on
the first of the two Guessing columns. Otherwise
enter an N in that column. Only when you enter
a Y in the first column do you refer to the Best
Match column. If there is a matching Y for that
question in the Best Match column, enter a Y in
the second Guessing column.
Total the number of Ys and enter those
numbers in the Totals spaces in the Guessing
columns for that exercise.
Situation 3
Situation 3 involves insufficient time. When you
have had insufficient time to consider all
questions on the exercise within the 35-minute
period, you indicate this fact by making two
marks on your answer sheet.
For example, suppose that you do not have
time to consider question 1.
Recording Times
Recording those answers where you had
insufficient time requires you to review both
your original answers and the Best Match
column. If you Xd a T on your answer sheet,
place a Y in the corresponding space on the first
of the two Time columns. Otherwise, enter an N
in that column. Only when you enter a Y in the
first Time column do you refer to the Best Match
column. If there is a matching Y for that question in the Best Match column, enter a Y in the
second Time column.
Total the number of Ys and enter those
numbers in the Totals spaces in the Time
columns for that exercise.
RECORDING
YOUR ANSWERS
www.petersons.com
LSAT Success
Guessing Position
OVERALL
Section 1
(Relationships)
Section 2
(Arguments)
Section 3
(Relationships)
Section 4
(Passages)
Section 5
(Arguments)
TEST POSITION
1
2
3
4
5
Total
Total
Total
Total
Total
Section 1
(Relationships)
Section 3
(Relationships)
Total Ys____
Total Ys____
Total Ys____
Total Ys____
Total Ys____
Total Ys____
Total Ys____
Total Ys____ 48 100____%
Ys____
Ys____
Ys____
Ys____
Ys____
Section 2
(Arguments)
Section 5
(Arguments)
Total Ys____
Total Ys____
Total Ys____ 48 100____%
Section 4
(Passages)
The final calculation immediately above expresses your overall position as a percentage of
questions for which you selected the best
answer. Simply speaking, the LSAT is designed so
that in order to get an average score, the
test-taker (you) needs to select slightly more than
60 percent of the best answers. The nature of
the LSAT scoring system is such that getting a
small number of best answers above 60 percent
results in a relatively large increase in the
placement of that score on the LSAT scale. The
higher the percentage of questions for which
you selected the best answer, the stronger your
position.
Total Ys____
Total Ys____ 28 100____%
Total Gs
100 = ____%
Total Ys
Relationships
LSAT Success
Total Gs
100 = ____%
Total Ys
Total Gs
100 = ____%
Total Ys
www.petersons.com
DIAGNOSTIC LSAT
Question-Type Position
Time Position
On the answer blanks, you marked every question
you did not have time to consider. To determine your
position with respect to the use of available time,
enter on the corresponding lines the total Ys from
the Time column for each exercise. Then complete
the calculations indicated.
Section 1
(Relationships)
Section 2
(Arguments)
Section 3
(Relationships)
Section 4
(Passages)
Section 5
(Arguments)
Section 1
(Relationships)
Section 3
(Relationships)
Section 1
(Relationships)
Section 2
(Arguments)
Section 3
(Relationships)
Section 4
(Passages)
Section 5
(Arguments)
Section 1
(Relationships)
Section 3
(Relationships)
Total Ys____
Total Ys____
Total Ys____
Total Ys____
Total Ys____
Total Ys____
Section 2
(Arguments)
Section 5
(Arguments)
Total Ys____
Section 4
(Passages)
Total Ys____
Total Ys____
Total Ys____
Total Ys____
Total Ys____
Total Ys____
Total Ys____
Total Ys____ 28 100____%
Total Ys____
Total Ys____ 28 100____%
www.petersons.com
Total Ys____
Total Ys____
Total Ys____
Total Ys____
10
LSAT Success
B-Match
(Y or N)
Guessing
(Y or N)
G-Match
(Y or N)
Time
(Y or N)
T-Match
(Y or N)
A B C D E G T
1. j j j j j j j
______
______
______
______
______
______
A B C D E G T
2. j j j j j j j
______
______
______
______
______
______
A B C D E G T
3. j j j j j j j
______
______
______
______
______
______
A B C D E G T
4. j j j j j j j
______
______
______
______
______
______
A B C D E G T
5. j j j j j j j
______
______
______
______
______
______
A B C D E G T
6. j j j j j j j
______
______
______
______
______
______
A B C D E G T
7. j j j j j j j
______
______
______
______
______
______
A B C D E G T
8. j j j j j j j
______
______
______
______
______
______
A B C D E G T
9. j j j j j j j
______
______
______
______
______
______
A B C D E G T
10. j j j j j j j
______
______
______
______
______
______
A B C D E G T
11. j j j j j j j
______
______
______
______
______
______
A B C D E G T
12. j j j j j j j
______
______
______
______
______
______
A B C D E G T
13. j j j j j j j
______
______
______
______
______
______
A B C D E G T
14. j j j j j j j
______
______
______
______
______
______
A B C D E G T
15. j j j j j j j
______
______
______
______
______
______
A B C D E G T
16. j j j j j j j
______
______
______
______
______
______
A B C D E G T
17. j j j j j j j
______
______
______
______
______
______
A B C D E G T
18. j j j j j j j
______
______
______
______
______
______
A B C D E G T
19. j j j j j j j
______
______
______
______
______
______
A B C D E G T
20. j j j j j j j
______
______
______
______
______
______
A B C D E G T
21. j j j j j j j
______
______
______
______
______
______
A B C D E G T
22. j j j j j j j
______
______
______
______
______
______
A B C D E G T
23. j j j j j j j
______
______
______
______
______
______
A B C D E G T
24. j j j j j j j
______
______
______
______
______
______
A B C D E G T
25. j j j j j j j
______
______
______
______
______
______
A B C D E G T
26. j j j j j j j
______
______
______
______
______
______
A B C D E G T
27. j j j j j j j
______
______
______
______
______
______
A B C D E G T
28. j j j j j j j
______
______
______
______
______
______
Ys____
Ys____
Ys____
Ys____
Ys____
Ys____
Ns____
Ns____
Ns____
Ns____
Ns____
Ns____
Section ____________
TOTALS
LSAT Success
11
www.petersons.com
Diagnostic Test
SECTION 1
TIME35
(A)
(B)
(C)
(D)
(E)
Questions 16
Eight businesses, A, B, C, D, E, F, G, and H, are
moving into a new eight-story building. Two
businesses will not locate on the same floor.
Each business will occupy one entire floor. The
floors of the new building are numbered one
through eight from the ground floor to the top
floor. Any plan for locating the businesses in the
building must meet the following requirements:
somewhere below
somewhere
somewhere above
somewhere
somewhere
two
three
four
five
six
two, seven
two, three
three, seven
one, two, seven
two, three, eight
(A)
(B)
(C)
(D)
(E)
24 QUESTIONS
MINUTES
four
five
six
seven
eight
13
DIAGNOSTIC TEST
11. If Ss line is busy and all of the other lines
are open, which of the following calls
CANNOT be made directly, through a
transfer, or through a series of transfers?
Questions 712
The First National Bank has five branches, S, T,
U, V, and W. The telephone system that connects the branches is not working properly. The
branch managers have determined that they can
place direct calls from their branches only to
certain other branches. They have further
determined that a branch can transfer calls to
any branch to which it can place a direct call
and can transfer calls only to such branches.
Direct calls can be made only from:
(A)
(B)
(C)
(D)
(E)
S to V
T to S and U
U to T
V to T
W to S and U
(A)
(B)
(C)
(D)
(E)
T only
U only
U and V only
T and W only
T and U only
(A)
(B)
(C)
(D)
(E)
T to V
S to T
V to S
U to V
W to T
www.petersons.com
U to V
U to S
V to U
V to S
W to V
Questions 1318
S
T
U
V
W
V to T
U to V
V to U
W to T
W to U
14
LSAT Success
SECTION 1
20. It could be true that Sarah arrived at
rehearsal
18
18
18
19
19
and
and
and
and
and
(A)
(B)
(C)
(D)
(E)
19
20
21
21
22
(A)
(B)
(C)
(D)
(E)
(A)
(B)
(C)
(D)
(E)
2
3
4
5
6
0..4
1..4
0..3
1..3
2..3
after
after
with
with
with
(A)
(B)
(C)
(D)
(E)
Nell, Sarah
Sarah, Ted
Sarah, Wendy
Nell, Sarah, Wendy
Nell, Sarah, Ted, Wendy
Questions 1924
(A)
(B)
(C)
(D)
(E)
LSAT Success
15
www.petersons.com
DIAGNOSTIC TEST
SECTION 2
TIME35
MINUTES
24 QUESTIONS
www.petersons.com
16
LSAT Success
SECTION 2
5. People who do not understand the laws of
probability often explain random happenings as the work of supernatural forces.
Those people would be much less likely to
believe in the supernatural if they had
knowledge of statistical probability.
LSAT Success
17
www.petersons.com
DIAGNOSTIC TEST
9. If they get customers to believe that earlier
patrons have given large tips, cosmetologists
can get larger tips from them. However, if
cosmetologists give the impression that they
are wealthy, their customers will not tip
them at all.
12. Even though art need not have an intellectual appeal, neither is it something that
immediately can be appreciated without
previous experiences that have deepened
ones capacity for appreciation. So it follows
that to get full enjoyment from art, people
must commit considerable time and effort
to developing their ability to observe and
understand art.
www.petersons.com
18
LSAT Success
SECTION 2
13. In France, as presidents have worked to
expand their personal power, they have
simultaneously achieved positive goals far
beyond their own and far beyond those of
their political parties. Well-executed
struggles for presidential power contribute
both to governmental effectiveness and
qualitative improvements in public policy.
Questions 1617
A sandhog claims that he has a list of thirty
deceased workers who worked as a group on a
tunnel that cut through a radon-intense area. He
started his list after an oncologist reported that
the incidences of cancer among sandhogs were
five times greater than the incidence of cancer
among other construction workers. The tunnel
contractor states that its records show that no
sandhogs working on the tunnel were exposed
to more radiation than was safe.
LSAT Success
19
www.petersons.com
DIAGNOSTIC TEST
17. Which of the following conclusions can be
drawn most reliably from the passage?
(A) the results from people not participating in the electoral process
(B) consequences of a consumers refusal
to purchase a product
(C) how citizens can effectively express
dissatisfaction with the two major
political parties
(D) why many citizens refuse to vote
(E) differences between the role of the
voter and that of the consumer
20. The author attempts to make her point by
(A)
(B)
(C)
(D)
(E)
Questions 1920
The excuse for not voting that is most commonly
given by citizens is that the two major political
parties do not offer competent candidates or
meaningful policies and positions. The nonvoting
citizen believes that by expressing dissatisfaction
as a consumer in the political process by not
casting his or her ballot, the parties will put
forward better candidates and positions. But this
misconceives the role of voter as that of consumer. In the marketplace, defection to another
product or failure to purchase a product sends a
powerful message that results in change,
improvement, and innovation. But not voting in
the political process will not produce a smorgasbord, but rather, will result in a menu with a
single entree and two side dishes.
www.petersons.com
constructing a simile
discrediting an analogy
making a circular argument
offering a unique hypothesis
presenting new evidence
20
LSAT Success
SECTION 2
24. Scientists have found through experimentation that baby female gorillas who were
nurtured by inanimate mother substitutes
that performed some parenting functions
were unable to function as mothers when
they had offspring. This teaches us that
infants should not be placed in the care of
babysitters and day-care centers but only
should be raised by their natural mothers.
LSAT Success
21
www.petersons.com
DIAGNOSTIC TEST
SECTION 3
TIME35
(A)
(B)
(C)
(D)
(E)
Questions 16
A chef is preparing sauces using eight different
ingredientsH, J, K, L, M, N, O, and P. According to the recipes, the following requirements
apply to the use of the ingredients:
(A)
(B)
(C)
(D)
(E)
(A)
(B)
(C)
(D)
(E)
H, J, K, O
H, L, M, P
L, M, N, P
K, L, M, O
M, N, O, P
J
L
N
O
P
www.petersons.com
2
3
4
5
6
Questions 712
(A)
(B)
(C)
(D)
(E)
H
J
K
L
P
(A)
(B)
(C)
(D)
(E)
H, L, P
J, N, M
K, L, N
K, M, P
L, M, O
(A)
(B)
(C)
(D)
(E)
24 QUESTIONS
MINUTES
22
LSAT Success
SECTION 3
8. If R is 5 miles from S, which of the following must be true?
(A)
(B)
(C)
(D)
(E)
S
S
S
S
S
is
is
is
is
is
closer
closer
closer
closer
closer
to
to
to
to
to
Questions 1318
Seven peopleK, N, O, P, Q, R, and Wown
condominiums in a four-story building.
P than to R.
Q than to R.
Q than to P.
P than to Q.
R than to P.
P is closer to Q than to R.
Q is closer to R than to P.
R is closer to P than to S.
R is closer to S than to Q.
S is closer to P than to R.
Q is due east of P
Q is 10 miles from P
S is at most 15 miles from P
S is at most 15 miles from R
S is due east of P
(A)
(B)
(C)
(D)
(E)
(A)
(B)
(C)
(D)
(E)
R, W
Q, R
P, W
P, Q, R
P, R, W
LSAT Success
23
two
three
four
two, three
three, four
www.petersons.com
DIAGNOSTIC TEST
20. Who must sit in the chairs on either side of
E if B sits next to D and if C sits next to F?
(A)
(B)
(C)
(D)
(E)
(A)
(B)
(C)
(D)
(E)
(A)
(B)
(C)
(D)
(E)
1
2
3
4
5
(A)
(B)
(C)
(D)
(E)
C and D
D and E
E and F
B and E
B and F
B
C
G
B, C
C, G
www.petersons.com
C
B
D
E
G
Questions 1924
(A)
(B)
(C)
(D)
(E)
B and G
B and C
B and F
C and D
C and G
(A)
(B)
(C)
(D)
(E)
24
LSAT Success
SECTION 4
SECTION 4
TIME35
MINUTES
45
50
Line
10
15
20
25
30
35
40
LSAT Success
55
60
65
70
28 QUESTIONS
25
www.petersons.com
DIAGNOSTIC TEST
2. According to the passage, which of the
following may NOT be limited by restrictions on the freedom of speech?
(A)
(B)
(C)
(D)
(E)
advertising claims
obscene films
amplified music in a public park
military secrets
political rhetoric
(A)
(B)
(C)
(D)
Line
10
15
20
25
30
35
www.petersons.com
26
LSAT Success
SECTION 4
40
45
50
55
60
65
(A)
(B)
(C)
(D)
(E)
aggravating
apologetic
indifferent
ironic
disapproving
LSAT Success
obscurity
realism
sensitivity
ambivalence
imagination
(A)
(B)
(C)
(D)
(E)
27
instinctual
antithetical
ambiguous
synonymous
suspect
www.petersons.com
DIAGNOSTIC TEST
13. Which of the following statements about
great novels, if true, would most strengthen
the authors assertions about literature and
fiction in the second paragraph?
30
35
40
10
15
20
25
45
50
55
www.petersons.com
60
65
70
75
28
LSAT Success
SECTION 4
15. The primary purpose of the passage is to
(A)
(B)
(C)
(D)
(E)
LSAT Success
approving
understanding
questioning
hostile
neutral
29
www.petersons.com
DIAGNOSTIC TEST
Line
10
15
20
25
30
35
40
45
50
55
www.petersons.com
60
65
70
30
LSAT Success
SECTION 4
23. The primary purpose of the passage is best
described as an effort to
(A)
(B)
(C)
(D)
(E)
LSAT Success
31
alarming
verifiable
unprecedented
implausible
sound
www.petersons.com
DIAGNOSTIC TEST
28. The author would most probably agree with
which of the following statements about the
relationship between business and laws that
encourage aspiration?
27. According to the author, the implementation of laws that encourage aspiration
would be difficult because
(A) evidence establishing a particular
businesss liability is not as likely to be
available
(B) information pertinent to each particular industry would have to be collected and analyzed
(C) the liability of a business would have
to be distinguished from the liability of
its employees
(D) businesses that are theoretical entities
can be easily disbanded
(E) the merits of conflicting, equally
worthy aspirations would have to be
considered
www.petersons.com
32
LSAT Success
SECTION 5
SECTION 5
TIME35
MINUTES
24 QUESTIONS
LSAT Success
33
www.petersons.com
DIAGNOSTIC TEST
4. Singularity saves politicians from having to
take infinite variations into account when
they establish rules and regulations for
diverse populations. A single set of rules
and regulations fits all.
7. Contrary to the spin placed on her reelection, representative Anthony did not achieve
a convincing victory. She points out that 60
percent of the voters cast ballots for her,
but a mere 25 percent of eligible people
voted for her. A substantial majority of
registered voters did not vote. Clearly, she
has the support of only a small minority of
the people she represents.
www.petersons.com
34
spin
convincing
majority
support
victory
LSAT Success
SECTION 5
10. If true, which of the following most weakens
the authors point of view?
Questions 910
LSAT Success
35
www.petersons.com
DIAGNOSTIC TEST
13. The Department of Agriculture will stop
inspecting milk processing plants because
no citations have been issued in the past
two years.
www.petersons.com
36
LSAT Success
SECTION 5
17. Federal courts are the sole interpreters of
the federal constitution. The federal
constitution requires that funding of the
federal courts is the responsibility of the
federal legislature. The courts and the
legislature are litigating a dispute over
funding of judges salaries. Judicial ethics
precludes a judge from involvement in
litigation that concerns that judges finances.
LSAT Success
37
www.petersons.com
DIAGNOSTIC TEST
21. For the first time in this century, the most
recent national census shows that towns
with populations under 10,000 are growing
more rapidly than cities of more than 1
million. This confirms that people prefer
human scale environments to rich, highdensity environments.
www.petersons.com
38
LSAT Success
QUICK-SCORE ANSWERS
QUICK-SCORE ANSWERS
Section 1
1. A
2. B
3. D
4. E
5. D
6. A
7. E
8. C
9. B
10. D
11. B
12. E
13. D
14. C
15. D
16. E
17. A
18. E
19. E
20. B
21. B
22. D
23. D
24. A
LSAT Success
Section 2
1. D
2. D
3. D
4. B
5. D
6. B
7. D
8. C
9. D
10. D
11. E
12. D
13. D
14. A
15. C
16. B
17. E
18. D
19. C
20. B
21. C
22. E
23. E
24. D
Section 3
1. C
2. A
3. C
4. E
5. D
6. C
7. D
8. C
9. C
10. E
11. C
12. B
13. D
14. B
15. E
16. C
17. B
18. B
19. E
20. E
21. B
22. D
23. D
24. C
Section 4
1. E
2. E
3. B
4. D
5. E
6. A
7. E
8. E
9. D
10. C
11. C
12. B
13. B
14. B
15. E
16. A
17. C
18. B
19. C
20. B
21. A
22. C
23. E
24. A
25. D
26. E
27. B
28. D
39
Section 5
1. D
2. C
3. C
4. D
5. C
6. D
7. D
8. C
9. E
10. C
11. E
12. B
13. E
14. C
15. A
16. D
17. E
18. B
19. C
20. E
21. D
22. C
23. D
24. A
www.petersons.com
DIAGNOSTIC TEST
EXPLANATORY ANSWERS
10. The correct answer is (D). U to V
In the following answer guide, the credited responses appear in bold type, and the visualization
that makes the credited response clear appears
before the question set. Use the visualization to
guide you in determining the credited answer.
Questions 1318
SECTION 1
Age
Questions 16
Youngest - Age 17
A, B, C, D, E, F, G, H
8
7
6
5
4
3
2
1
A, B, C, D, E, F
Age 22
Oldest
F Between
D
13. The correct answer is (D). Fern is 18
years old.
Questions 1924
Arrivals
N, P, R, S, T, W
Earlier
Later
Questions 712
S, T, U, V, W
www.petersons.com
40
LSAT Success
EXPLANATORY ANSWERS
7. The correct answer is (D). Numerous
kinds of psychological stress produce similar
physiological symptoms.
PointPeople undergo stress-induced
physiological changes when they lie.
IssueExtension question/weakening
evidence
SECTION 2
In the following answer guide, the credited
responses appear in bold type and the guide that
directs you toward the credited response appears
within the answer choice context. The first
reference is to the point of the argument and the
second is to the nature of the issue involved.
IssueExtension question/conclusion
IssueExtension question/weakening
evidence
LSAT Success
41
www.petersons.com
DIAGNOSTIC TEST
Questions 1920
Questions 1617
16. The correct answer is (B). All thirty of
the deceased sandhogs died of radiationrelated cancer.
PointThirty dead sandhogs were exposed
to radiation that the contractor
claims was safe.
IssueExtension question/weakening
evidence
www.petersons.com
42
LSAT Success
EXPLANATORY ANSWERS
SECTION 3
Questions 1318
Condos
K, N, O, P, Q, R, W
3
2
1
N
K
Q+N
Questions 16
Ingredients
H, J, K, L, M, N, O, P
M must N
If
Then
no
K and P
H
J
O
Cannot be together
Questions 1924
Seating
B, C, D, E, F, G
Questions 712
P, Q, R, S
B
20
R
R
next to
not next to
P
10
not next to
LSAT Success
43
www.petersons.com
DIAGNOSTIC TEST
SECTION 4
1. The correct answer is (E). More advertising will be protected by the First Amendment.
Paragraph 1/Sentence 5
3. The correct answer is (B). Price advertising is protected by the First Amendment.
Paragraph 2/Sentence 8
25. The correct answer is (D). The capabilities of different business vary and such
variation is not taken into account by laws
that encourage aspiration.Paragraph 2/
Sentence 6
www.petersons.com
18. The correct answer is (B). The government could get better technology than it
does.Paragraph 2/Sentence 1
44
LSAT Success
EXPLANATORY ANSWERS
5. The correct answer is (C). Education
includes the cultivation of creative mental
abilities.
PointEducation should include instruction
in the use of information taught.
IssueExtension question/conclusion
SECTION 5
Questions 910
9. The correct answer is (E). demonstrate
that the alleged problem is actually a
pseudoproblem
PointEmployment of people who are 16+
is at a record high despite a minimum wage.
IssueDescription question/tactic
LSAT Success
45
www.petersons.com
DIAGNOSTIC TEST
12. The correct answer is (B). deflects the
question by describing the number of
women employees
www.petersons.com
46
LSAT Success
R E D A LERT
PREPARATION ANXIETY BRINGS US TOGETHER
DO
PREPARATION
BARRIER?
RED
IS
47
ALERT
RED ALERT
window. The hair stands up on the back of your
neck, and your heart begins to race. You have
haunted-house syndrome. Imagination takes over
and projects horrors of every description onto
the LSAT, and you convince yourself the score
will be a measure of your worth as a person and
reveal your inadequacies to the world.
You are in needless pain. Do not project
your fears onto the LSAT and give the syndrome
a chance to undermine your removal of the
barriers between you and your best performance.
Sure, the LSAT is the most important factor in
law school admission, but it does not measure
your worth or predict your success in life. It
simply does not merit projecting any such
meaning onto it.
WHAT
CAUSES PREPARATION
ANXIETY?
Most people find themselves behind the preparation-anxiety barrier. To begin the barrier-removal
process, you recognize its existence. Do not deny
it or try to rationalize it. Accept your feelings.
Acceptance is essential to your removing
your anxiety barrier. You need to use your anxiety
to power your understanding of the LSAT. Understanding the LSAT will give you the control you
need to perform your personal best. And the best
way to begin your understanding is to gain control
over your anxiety through training.
www.petersons.com
RED
48
ALERT
LSAT Success
RED ALERT
about the LSAT should motivate you to explore
effective test-taking techniques and to implement
a personal training plan, and it is through these
things that you gain control over the test.
However, how you worry is very important if the worry experience is to be positive and
not produce a barrier to performance or undermine your training program. The essentials of
effective worrying are: identifying the threat,
exploring the options available for dealing with
it, and installing a safety net.
Practice worrying!
Worry practice is useful and effective. Throughout
your life, people have encouraged you to overcome barriers with the words dont worry. As
you face the LSAT, your parents, friends, spouse,
and others assure you that Everything will be OK.
Dont worry, you always do well. Accept such
encouragement, but do not mistake it for positive
thinking or a good training practice.
Worry about the LSAT.
Irrespective of how often you are told not to,
you will worry about the LSAT. And, perhaps
ironically, its a good idea. Constructive worrying
LSAT Success
RED
49
ALERT
www.petersons.com
RED ALERT
Do relaxation and
concentration help reduce
anxiety and keep that
preparation barrier down?
www.petersons.com
RED
50
ALERT
LSAT Success
Success Session 1
ACADEMIC CONDITIONING MAY BE HAZARDOUS
TO YOUR LSAT SCORE
convinced. And unless I can convince you that
your academic conditioning must be put aside,
your test training and LSAT performance will
suffer, so here we go.
CAN
ISNT
OTHER
LSAT
TEST?
THE
PROWESS?
Academic success is probably a major contributor
to your interest in law school and the legal
profession that have, in turn, led you to the
challenge of the LSAT. You probably have some
confidence in your academic ability. You have
learned that a superior academic performance
depends upon superior conditioning and study
and test-taking techniques. Therefore, you reason
that the conditioning and techniques that have
produced good academic results for you in the
past should also produce a superior LSAT score.
For your reasoning to be correct, the LSAT
should be a form of academic performancebut
this is not the case.
THE LSAT IS NOT AN ACADEMIC EXERCISE.
For this reason, relying upon academic conditioning and techniques may actually place barriers
that impair rather than enhance your LSAT
performance. By training to circumvent these
barriers, you can dramatically improve your score
potential.
AVOIDING EFFECTIVE ACADEMIC
TECHNIQUES CAN ACTUALLY IMPROVE
YOUR LSAT SCORE.
SKEPTICAL?
Sure you are. Skepticism is one of those successful academic techniques. You have been conditioned to question, and you insist on being
51
SUCCESS SESSION 1
DONT SUBSTITUTE THE COMFORT OF
APPARENT STUDY FOR EFFECTIVE LSAT
TRAINING.
WHAT
LSAT
BarrierConvert the
Unfamiliar into the Familiar
When faced with the unfamiliar, we tend to
respond by doing something familiar and
comfortable. Thus, the majority of those facing
the LSAT enroll in a course that they believe
will teach them the substantive information
required for the testbut, as we have just seen,
the LSAT requires no substantive information.
As a substitute for effective preparation,
weeks are wasted in the familiar classroom context
searching for a way to convert the LSAT into the
familiar (and comfortable?) academic test. The
search is futile. It is a barrier that does little more
than generate more preparation anxiety. Those
who recognize the futility of traditional classroom
preparation often attempt to sharpen their
academic test-taking techniques by slogging
through hundreds of LSAT questions. But this type
of practice reinforces techniques that, when
applied, likely constrain LSAT performance.
www.petersons.com
BarrierComplex Drives
Out Simple
Your academic conditioning discourages choosing the simple or superficial answer to a test
question and encourages you to search for the
complex and thorough. When this conditioning
is projected onto the LSAT, the test-taker
frequently selects a less-than-appropriate answer
because the best answer seems too simple.
Dont bypass the simple LSAT answer choice just
because it would not be a strong response on an
academic test.
52
LSAT Success
BarrierReading for
Information
In an academic context, reading is the primary
way to acquire information. When the LSAT
presents a reading passage, the conditioned
reaction is to read it for the information it
contains. But the passages you will find in the
LSAT are not there to present information that
you must commit to memory for future use. Such
memorization would only detract from the
purpose of LSAT reading passages, which is to
provide a familiar vehicle through which your
reasoning skill might be measured.
The Completion-Delusion
Barrier
Academic conditioning promotes the delusion
that you can enhance your test performance by
considering every question and answering every
question to the extent that time allows. For
virtually everyone who takes the LSAT, the
opposite is true. No points are gained by
simply completing the entire LSAT. Your
score results exclusively from the number of
correct answers you select. In fact, it is by not
completing the test that most people will reach
their top test score.
BarrierPersuaded by Logic
WHAT
TEST-TAKING TECHNIQUES
LSAT
The Question-Is-the-Question
Barrier
LSAT Success
53
www.petersons.com
SUCCESS SESSION 1
MAKE NO ASSUMPTIONS WHEN TRYING TO
SELECT THE BEST ANSWER TO AN LSAT
QUESTION.
BarrierHard Questions
Count More
www.petersons.com
54
LSAT Success
BarrierFair Means
Unpredictable
Finally, your previous academic conditioning
promotes the notion that a fair test is one that is
unpredictable. If you know and understand the
substantive information presented in the academic course, you can do well in responding to
any question that may appear on a test. If the
questions are any more predictable than this, the
test is not perceived as fair in the sense that a
person can do well by knowing only the
information to which the test is limited. If the
test deals only with the material in one lecture of
a thirty-lecture course, it is very predictable and
not likely to be perceived as fair by most
students. So it should follow that for the LSAT to
be fair, as it must be in order to be effective in
deciding the admissions of students to law
school, it should also be unpredictable.
The idea of its unpredictability is reinforced
by the point made earlier in this session: The
LSAT does not measure the subjects customary to
academic or other standardized tests, such as
facts, vocabulary, grammar, mathematical
formulae, and the like. But, as you have seen
elsewhere in this session, the LSAT reality is
often the opposite of what you might expect,
and the matter of its predictability is no exception. The LSAT is totally predictable. Each of the
subsequent success sessions provides techniques
that can be used to take advantage of this
predictability.
As noted earlier, the LSAT is designed to
differentiate among test-takers. This differentiation cannot be left to chance by the test-maker.
The differentiation must be totally predictable.
Without this predictability, the test would not
serve its purpose. So, the test is designed to
consistently produce the same, predictable
results. If you think or believe otherwise, you
LSAT Success
55
www.petersons.com
SUCCESS SESSION 1
will confront a barrier to your best performance.
To avoid this barrier, treat the LSAT as a ruler. It
remains constant so that the measurement results
are consistent and predictable. Each edition of
the test is the same. Each question set measures
the same things in the same wayover and over.
www.petersons.com
56
LSAT Success
Success Session 2
BECOME FAMILIAR WITH THE LSAT
FAMILIARITY
BREEDS SUCCESS!
WHAT
WHAT
IS THE JUSTIFICATION
FOR THE
LSAT?
LSAT?
57
SUCCESS SESSION 2
HOW
DOES THE
LSAT
WORK?
Directions
Each section comes with directions. The directions tell you what you are expected to do on
that section. Directions are important.
Statements
A statement precedes most question and answerchoice sets. The statement provides certain
information, some of which is relevant to the
questions and answer choices that follow the
statement. It is easy to confuse statements and
questions.
Questions
BASIC ELEMENTS OF
Passages28 questions
Relationships24 questions
Arguments24 questions in each of two
Arguments sections
Sections
While they are commonly referred to as test
sections, these elements are, in fact, separate tests.
It is important to understand that each LSAT
consists of five of these tests, as well as a Writing
Sample. Although there is only one LSAT score,
there are five different tests on every LSAT.
www.petersons.com
Answers
Each LSAT question is followed by five answer
choices or options. This means that there are
some 500 answer choices on every LSAT.
58
LSAT Success
LSAT Success
59
www.petersons.com
SUCCESS SESSION 2
www.petersons.com
60
LSAT Success
LSAT Success
61
www.petersons.com
SUCCESS SESSION 2
WHAT
Time
DOES THE
LSAT
PERFORM?
The LSAT has difficulty as it strives to successfully meet its stated objective of ranking applicants to law schools so that the very skilled and
superskilled are meaningfully differentiated for
admission selection. In terms of actual performance, the LSAT does not meet its ranking
objectives very well. For example, rather than
distributing test-takers across the score scale, the
LSAT tends to bunch test-takers at various places
on the scale. The test maker explains this
phenomenon variously.
Whatever the explanation, you will design
your LSAT performance strategy to take advantage of this tendency in the test. It is a strategy
that can produce the best test result for you.
www.petersons.com
LSAT
HOW
Number
of Wrong
Answers
7
18
21
25
29
32
37
62
% of
Test-Takers
with More
Wrong Answers
% of
Test-Takers
with Fewer
Wrong Answers
99.9
95
90
80
70
60
50
0.1
5
10
20
30
40
50
LSAT Success
WHAT
LSAT
HOW
LAW SCHOOL
LSAT Success
DOES THE
LSAT
INFLUENCE THE
REJECTION
DECISION?
63
www.petersons.com
SUCCESS SESSION 2
FOR EACH LAW SCHOOL TO WHICH YOU
APPLY, YOUR ADMISSION INDEX IS A
FUNCTION OF YOUR LSAT SCORE AND
YOUR GPA.
WHAT
DOES THE
LSAT
MEASURE?
www.petersons.com
64
LSAT Success
LSAT Success
65
www.petersons.com
Success Session 3
YOUR STRATEGY IS ESSENTIAL TO YOUR SUCCESS
YOUR
WHAT
WHAT
WITH A STRATEGY.
IS THE
STRATEGY?
LSATS
PREPARATION
IS YOUR PREPARATION
STRATEGY?
67
SUCCESS SESSION 3
WHAT
WHAT
WHAT IS MEANT BY
TEST-PERFORMANCE FACTORS?
Your level of anxiety, academic test-taking habits
and mechanics, reasoning skill, time use, and
self-discipline are the major factors that influence
performance on the LSAT. Facts, perspectives,
and other such information are of very little
consequence. The training in this book is
designed to minimize the impact that the factors
of anxiety and entrenched academic test-taking
habits and mechanics have on your performance.
By learning and applying procedures that take
advantage of the LSATs nature, you will be able
to avoid any negative impact these performance
factors might have on your test score.
The reasoning skill required by law schools
is a critical performance factor on the LSAT. For
most people it is the most important factor by
far. Your training regimen should include regular
monitoring of this skill as you refine it, and your
test plan should be modified to take improvement in your reasoning into account.
Self-discipline is the most difficult of all
performance factors to build into your plan. Only
you know if you are strict or lax in following the
test-taking procedures you train to use. Avoid
being overly optimistic in this regardthere is a
great tendency to believe that each of us enjoys a
generous measure of self-discipline, but the
evidence consistently proves the contrary. If
self-discipline, sometimes referred to as concentration, persistence, focus, or effective time-
www.petersons.com
IS AN OPTIMAL MIX?
The LSAT establishes an optimal mix of performance factors to ensure that the test results are
precisely those desired; for example, that the
proportion of test-takers that will produce a
score of 150 satisfies the number that is the
objective for the test. Among other things, this
effort leads to the structuring of the LSAT as
separate tests, the inclusion of a specific number
of questions in each of the tests, the selection of
certain types of questions, the allocation of time
for each test, and the following of a particular
presentation pattern. To achieve an optimal mix,
many experiments with the LSAT variables are
conducted, increasing this, decreasing that,
changing something else. Once the variables
produce exactly the results intended, the test is
ready to be administered.
As with the LSAT strategy, your optimal
mix of performance factors is the one that best
ensures that your objectives are achieved. Just
enough worry to make sure that you are in
control of the training and testing process is
best. Balancing your speed of working on the
questions against the accuracy of your answers
so that you select the expected number of
wrong answers is crucial. Mechanics are importantthe sequence in which you consider
questions, the order in which you examine
answer choices, the recording of your answers in
the test book, the transcription of answer
choices onto the answer sheet, and others.
HOW
ABOUT CONSISTENT
PROSECUTION?
As you will understand better when you learn
more about the structure of each of the question
types on the LSAT, it is difficult for the test itself
to perform consistently. It is also difficult for you
to perform consistently over the course of the
205-minute LSAT. Yet, your best performance
will depend on consistency.
68
LSAT Success
WHERE
TEST PLAN?
You will recall that the LSAT consists of five
separately timed tests and a Writing Sample. Each
of the five tests consists of four sets of questions
and answer choices. Thus, the LSAT is made up
of twenty of these question/answer sets. A good
way to visualize this is to think of the LSAT as
consisting of twenty minitests of six or seven
questions each.
THE QUESTION/ANSWER SET IS THE FOCUS
OF YOUR TEST PLAN.
WHAT
MAKES UP THE
MINITESTS?
LSAT
LSAT Success
As you train, the optimal mix of your performance factors will change, and, as the optimal
mix changes, so will your plan.
69
www.petersons.com
R E D A LERT
THE 9-12-18 TEST-PLANNING SYSTEM
YOUR
TEST-PLANNING PROCESS
STARTS HERE.
WHAT
IS THE
9-12-18
SYSTEM?
RED
CONSIDER THE
CAREFULLY.
FOLLOWING EXAMPLE
71
ALERT
RED ALERT
onto the entire section-test, were you to perform
on three question sets with the same accuracy as
you did on the one.
The third formula shows what happens if
you decide to work on only two question sets
during the time allocated for the section-test.
This means that there is an average of about 18
minutes (18 2 = 36) available for each set. The
number of best answers (6) is multiplied by 2 to
project your one-set performance onto the entire
section-test, were you to perform on two
question sets with the same accuracy as you did
on the one set.
The third number in the formula represents
the credited or best answers that will result from
guessing on question sets that you do not
otherwise consider. Recall that you should
always guess on an LSAT question, even when
you have not considered it. Since there are five
answer choices to every question, a guess has a
one-in-five chance of being the best answer.
There are six questions per set on the Relationships section-test; consequently, over a sixquestion set, chance alone will result in your
selecting one best answer, even though you give
that question no other consideration.
Obviously, if you consider all four question
sets in the section-test, you get no guessing
bonus. If you consider three of the four question
sets, you get a guessing bonus of one. If you
consider two of the four question sets, you get a
guessing bonus of two. This results from the fact
that, even though you have not considered a
particular six-question set, you have arbitrarily
selected (guessed) an answer for each of those
questions.
The total of best answers, the fourth
number in the formula, shows how you would
perform on the whole section-test, were you to
consider and complete four, three, or two
question sets. In the example, you can see that
you will select the fewest wrong answers on the
section-test if you consider only two question
sets and guess your answers for the other two.
This pretest process produces your optimal
balance between speed and accuracy on the
Relationships section-test of the LSAT. The
example shows that by considering all four
question sets (a total of twenty-four answers),
you can plan on selecting sixteen (twenty-four
total answers minus eight best answers) wrong
answers. By considering just three question sets,
you can plan on selecting fourteen wrong
answersnot a large difference. However, by
WHAT
24+0= 8
3 3 + 1 = 10
6 2 + 2 = 14
www.petersons.com
RED
72
ALERT
LSAT Success
RED ALERT
Relationships Section-Test
9 minutes
24+0= 8
12 minutes
3 3 + 1 = 10
18 minutes
6 2 + 2 = 14
Arguments Section-Tests
(remember, there are two
Arguments section-tests)
9 minutes
3 4 + 0 = 12 2 = 24
12 minutes
6 3 + 1 = 19 2 = 38
18 minutes
6 2 + 2 = 14 2 = 28
Passages Section-Test
9 minutes
12 minutes
18 minutes
3 4 + 0 = 12
5 3 + 1 = 16
5 2 + 2 = 12
HOW
2 sets
each
3 sets
each
3 sets
each
3 sets
each
STEP 1
at 18 minutes
at 12 minutes
at 12 minutes
at 12 minutes
LSAT Success
RED
YOUR OWN
STEP 2
Next, give yourself an additional 3 minutes. Go
back to the set, and answer any questions you
did not get to in the first 9 minutes, or review
the questions you answered in the first 9
minutes, and change your answers if you wish.
73
ALERT
www.petersons.com
RED ALERT
STEP 5
STEP 3
WHATS
STEP 4
Turn to the appropriate Quick-Score Answers
section for the question-sets used in the minitests
(page 39 for the Diagnostic LSAT, page 191 for
Practice Test 1, and page 229 for Practice Test 2).
For each set, determine the number of best
answers you selected in the 9-minute set
(circled), 12-minute set (boxed), and 18-minute
set (asterisked). Enter those numbers on the
Pretest Performance Tracking Worksheet on
page 75. Alternatively, you can use the following
calculation form.
www.petersons.com
RED
NEXT?
________ sets
________ sets
________ sets
74
ALERT
LSAT Success
RED ALERT
75
ALERT
Circles (9 minutes) x 4 + 0 =
Boxes (12 minutes) x 3 + 1 =
Asterisks (18 minutes) x 2 + 2 =
Largest Total
From 4, 3, or 2 sets?
Arguments
Minitest
Circles (9 minutes) x 4 + 0 =
Boxes (12 minutes) x 3 + 1 =
Asterisks (18 minutes) x 2 + 2 =
Largest Total
From 4, 3, or 2 sets?
Passages
Minitest
Circles (9 minutes) x 4 + 0 =
Boxes (12 minutes) x 3 + 1 =
Asterisks (18 minutes) x 2 + 2 =
Largest Total
From 4, 3, or 2 sets?
LSAT Success
RED
www.petersons.com
Success Session 4
LSAT QUESTION SETS
WHAT
IS THE BASIC
COMPONENT?
LSAT
WHAT
IS THE
PROFILE?
LSAT
HOW
DESCRIBED?
In the recent past, LSAT answer choices have
appeared in two formats: the single-option
format and the multiple-option format. Only the
single-option format has been used on the most
recent tests. It is very unlikely that a multipleoption answer choice will appear on an LSAT
that you take. However, a brief description of the
multiple-option answer choice is included here.
If the time available to you permits, read through
the description. In any event, do not spend any
substantial time with the multiple-option
answer-choice description.
QUESTION
77
SUCCESS SESSION 4
(A)
(B)
(C)
(D)
(E)
I only
II only
I and II only
II and III only
I, II, and III
WHAT
REQUIRED BY THE
LSAT?
www.petersons.com
78
LSAT Success
CAN
ANSWER CHOICES
(A) It is a deciduous tree.
(B) It is an acorn tree.
(C) It is a hardwood tree.
(D) It is an oak tree.
(E) It is an unknown species of tree.
LSAT Success
79
www.petersons.com
SUCCESS SESSION 4
DIRECTIONS
Answer the following question as quickly
as you can. Circle the best answer choice.
STATEMENT
The card-master holds four cards in a
stack. Each card has a single letter on the
front side and a single number on the
back side. The master states that if a card
has a vowel on one side it will have an
even number on the other side. The
master places the four cards on a table
with the visible sides showing A, B, 1,
and 2, respectively.
QUESTION
2. Of the following, which names only
the cards that must be turned over to
determine whether what the master
said is true?
WHAT
ANSWER CHOICES
(A) The cards showing A and B.
(B) The cards showing A and 1.
(C) The cards showing A and 2.
(D) The cards showing A, B, and 1.
(E) All four of the cards.
A method for dealing with the four cards on the
table can be depicted as follows:
IS OBFUSCATION?
Obfuscation is the LSATs primary tool. Obfuscation makes it difficult for the test-taker to see the
basic reasoning tasks in most LSAT questions.
The greater the degree of obfuscation, the more
difficult the selection of an LSAT answer choice
becomes for the test-taker. Nevertheless, the
basic reasoning task is thereit is the essence of
every LSAT question. In fact, the basic reasoning
task must be there if the test is to measure your
skill in dealing with it. With some training, you
can become effective at locating the basic task,
avoiding the academic conditioning the problem
sets out to exploit, and selecting the best
answers consistently.
In this next example of the basic reasoning
task, you are asked to answer a question about
cards. Give it a try, and see if you can notice how
the LSAT could obfuscate your route to the best
answer. Once again, the basic elements of the
problem have been identifiedjust remember,
they wont always be spelled out in this way.
www.petersons.com
80
LSAT Success
LSAT Success
WHAT
OBFUSCATION TACTICS?
Three of the five general types of obfuscation
tactics used on the LSAT involve varying the
presentation of the basic task. First, the context
in which the basic task is presented is varied. For
example, three seemingly different section-tests
are usedArguments, Passages, and Relationships. Within each of these section-tests, the
appearance of the questions is varied to add to
the impression that many different tasks are
required. The question structure is emphasized in
the mind of the test-taker, who tends to concentrate on differences in form and overlook the
essential similarity of LSAT questions. Varying the
presentation of the basic task causes the LSATs
focus to seem different from what it really is.
Related to the obfuscation-by-variation
tactic is the use of excess or excessive information. Although it is employed elsewhere on the
test, this tactic is seen best in the Passages
statement, which gives the test-taker much more
information than is needed to respond to the
question-answer choice sets. The test-taker
concentrates on the information presented,
trying to understand and retain the data by using
processes that have proven effective in past test
experiences. Thus, the test-taker becomes
absorbed in processes that detract from work on
the real LSAT task.
The third obfuscation tactic involves the
omission of information in the statement and
question. The test-taker gets the impression that
there is insufficient information to respond to the
question, and this missing data grabs and keeps
the test-takers attention away from the basic
reasoning task involved. While it often may
appear otherwise, the information needed to
respond to the question is always givenwhat is
not stated is often as informative as what is
stated.
These three tactics generally create
difficulties with question location for the
test-taker. The test-taker uses limited test time
seeking rather than responding to the question.
You can minimize the impact of these tactics by
remembering that the basic task of reasoning
81
www.petersons.com
SUCCESS SESSION 4
The invited question is also very difficult
for the well-conditioned test-taker to resist. The
invited question obfuscation tactic often shows
up on television quiz showsthe host reads only
a few words of the question and a contestant
blurts out an answer. A more subtle form of the
same thing was involved in the card-master
example. Although the question invited you to
identify cards that might confirm the truth of the
card-masters statement, the operative question
required you to identify only those cards that
could disprove the truth of the statement. The
question asked was not the operative question.
You will find that nearly all LSAT questions are
generic and unclear. Consequently, they invite
interpretation. Effective interpretation of the
questions best results from a review of the
answer choices. Seldom can interpretive data be
found other than in the answer choices.
www.petersons.com
82
LSAT Success
Success Session 5
RELATIONSHIPS PROBLEMS
WHAT
ARE
QUESTIONS
RELATIONSHIPS
ABOUT?
Of the three LSAT question formats, Relationships questions present the basic task most
obviously. This clarity can assist you in developing the skills and procedures you need for
handling the basic task in the other two LSAT
problem formats. Note also that of the three
question formats, Relationships is also the most
difficult for most people to master. Thus, we
begin specific problem-type training with
Relationships. Working on Relationships first
gives you additional time to get the basic task in
this context under control and thus, helps to
maximize your test performance.
WHY
SO
RELATIONSHIPS
DIFFICULT?
ARE
IN
QUESTIONS
OF A
83
SUCCESS SESSION 5
HOW
DO
WORK?
RELATIONSHIPS
QUESTIONS
The Directions
The directions for Relationships questions are
brief and not very enlightening. The LSAT tells
you that questions are based on a set of conditions and that you are to select the response that
most accurately and completely answers the
question. And thats it! The important thing you
are not told is that the conditions that appear in
the statement apply to all questions, and the
conditions that appear in a question apply to
that question only. As you will soon learn, the
response that most accurately and completely
answers a question will be the only answer
option that satisfies all of the conditions applicable to that question. There is no ambiguity to
resolve when dealing with Relationships problems.
The Statement
Relationships statements usually consist of two
parts, designated here as facts and rules. Although the facts and rules are short and seemingly simple, there is a possible complication
unique to Relationships questions. While each
question in a set shares the common statement,
some problems include an additional statement
embedded in the question itself. In general,
about two thirds of Relationships questions
include one of these supplemental statements.
Supplemental statements can also overrule or
disable a fact or rule that appears in the common
statement. Be alert!
The facts and rules in the common statement apply to all questions in the set. The facts
and rules in a supplemental statement apply only
to the questions in which they appear.
The Question
The question circumscribes or brackets the
answer selection to be made. As described
previously, it also may add facts and rules to
those that are common to the entire question
set, further narrowing the relevant conditions
with which you must deal. However, the
question seldom reduces the relevant conditions
to one.
Relationships questions are very straightforward. There are only three basic questions.
Which of the following must satisfy the
conditions/be true?
Which of the following could satisfy the
conditions/be true?
Which of the following cannot satisfy the
conditions/be true?
www.petersons.com
84
LSAT Success
RELATIONSHIPS PROBLEMS
The
Question
Which of the
following
must?
Which of the
following
CANNOT?
Which of the
following
could?
If this is true,
which of the following
must be true?
If this is true,
which of the following
could be true?
If this is true,
which of the following
CANNOT be true?
All of the
following must
be true EXCEPT?
All of the
following could
be true EXCEPT?
All of the
following CANNOT
be true EXCEPT?
The Answers
The LSATs selection of answer options gives you
the wrong-answer advantage. Not only are there
just five answers from which to select, but four
of them do not satisfy one or more of the
conditions required by the problem. As you
learned previously, it is difficult for the LSAT to
produce attractive wrong answers, since every
wrong answer fails to satisfy one or more of the
conditions required by the combined statement
and question. The LSAT attempts to disguise this
weakness, but, for most answer options, it is
relatively easy to detect. In fact, for most
LSAT Success
85
www.petersons.com
SUCCESS SESSION 5
Since the question asks for that which must be
true, wrong answers will be those that could or
cannot be true.
Answer option (A) overstates what must be
true. The supplemental statement requires that
no more than two of the three members of a
committee be shared with another committee.
Thus, all three committee members cannot be
shared by two committees.
Answer option (B) understates what must be
true. The common statement requires that one director be a member of both the Audit and Planning
committees. Thus, it cannot be that no committee
members are shared by the two committees.
Answer option (C) overstates what must be
true. The common statement requires that one
member of the Audit and Finance committees be
shared, not two. The supplemental statement
requires that no more than two members of a
committee be shared with another committeeit
does not require that two members be shared.
Thus, it could be that two committee members
are shared by two committees, but it is not
necessarily so.
Answer option (D) both understates and
overstates what must be true. The common
statement does not require that the member of
the Audit, Finance, and Planning committees be
the same person who is shared by the Audit,
Finance, and Shareholder Relations committees.
It is possible but not required. The supplemental
statement requires that no more than two
members of a committee be shared with another
committee. It does not require that any members
be shared. Thus, it could be that the Planning
and Shareholder Relations committees have only
one member in common, but it need not be so.
Answer option (E) illustrates a variation of
the too-little-too-much technique sometimes
used by the test-maker. It is wrong because it is
far too precise to meet the stated conditions.
This technique can be disconcerting and
distracting to a test-taker who fails to recognize it
and tries to develop some conclusive proof.
By identifying a condition implicit in the
answer option that fails to satisfy the question,
you can determine that an answer is wrong and
dump it. The wrong-answer patterns and
obfuscatory tactics are summarized below.
For must questions, wrong answer options
are those that could or cannot satisfy required
conditions, even though they use
www.petersons.com
86
LSAT Success
RELATIONSHIPS PROBLEMS
For could questions, wrong answer options are
those that cannot satisfy required conditions,
even though they use
HOW
LSAT
QUESTIONS?
DOES THE
DIFFICULT
CREATE MORE
As you have seen, making more difficult questions is not simply a matter of asking tougher
questions. Rather, in order to create more
difficult questions, the LSAT must develop better
answers. The LSAT wants the substantial majority
of questions on any section-test to be of medium
difficulty, with some 4060 percent of test-takers
selecting the best answer. It wants a smaller
proportion of questions to be relatively easy,
with more than 60 percent of test-takers selecting the best answer. And it is important that an
even smaller proportion is to be relatively tough,
with fewer than 40 percent of test-takers
selecting the best answer.
To adjust the difficulty level of a question
or question set, the LSAT use two basic techniques. First, the complexity of the relationships
can be varied. This is demonstrated in the
preceding example by the inclusion of the
supplemental statement, which defines the size
of the committees and limits the nature of their
composition.
HOW
COMPLEXITY OF
QUESTIONS?
RELATIONSHIPS
LSAT Success
87
www.petersons.com
SUCCESS SESSION 5
HOW
VISUALIZATION TECHNIQUE?
First, you read the facts and rules. It is rare that
these alone will provide sufficient information for
you to develop an effective visualization, so you
must next scan the questions. At this point, the
core of information you will be working with
becomes clear, and you can begin your
visualization.
You usually will find some space to record
your visualization on the pages of the question
set, but occasionally you will encounter a full
page with no white space. Scratch paper and
shirt cuffs are forbidden. What to do? Just fold
the preceding or succeeding page in half, and
use the space thus exposed.
In the LSAT, relationships are defined in
relative rather than specific terms. You will find
such statements as Mary is older than John and
John is younger than Betty. You will not read
that Mary is 10, John is 8, and Betty is 12. The
most efficient visualization must anticipate this
lack of specificity and allow for the shifts that
inevitably will be required by the questions.
You must remember that the lines in your
visualization do not depict exact relationships
they are more akin to rubber bands, capable of
www.petersons.com
88
LSAT Success
Success Session 6
ARGUMENTS AND PASSAGES PROBLEMS
WHAT
OF
WHAT
THE STATEMENTS?
(A)
(B)
(C)
(D)
(E)
The most apparent difference between Arguments and Passages statements is their format.
Passages statements are long and heavy on detail.
They contrast with Arguments statements, which
are short and offer little detail. These differences
89
twentieth-century politicians
frustrated modern inventors
overpaid factory workers
precursors of todays scientists
nineteenth-century economists
SUCCESS SESSION 6
The extension question asks you to select the
answer option that best represents an extension
of the conditions set out in or required by the
statement. An example of the extension question
type follows.
HOW
STRUCTURED?
The questions and answer options for Arguments
and Passages are presented in three formats. The
first is the question-and-answer format with
which you are likely to be very familiar. As the
name implies, a question is posed to which the
answer options are responses. The second format
is the question-question format, in which the
question asks you to select the question that is
answered by the information given or implied by
the statement. The last format is sentence
completion. The question is the first part of the
sentence that the answer options complete in
various ways. There is no question mark involved
in this question format.
The variety of formats serves to obfuscate
the question. The formats are summarized in the
illustration on page 91. Although they are
included in order to leave nothing to chance, the
multiple-option format is very unlikely to appear
on the LSAT and you should spend only a
minimal amount of time considering it.
www.petersons.com
90
LSAT Success
LSAT Success
91
www.petersons.com
SUCCESS SESSION 6
answer patterns for the three basic question
types, you will be positioned to take full advantage of the wrong-answer options to every
Arguments and Passages question.
Take a moment to recall that the bestanswer option is the only one that satisfies all of
the relevant conditions set out in the statement
and question. When you identify a question as a
variation of Which of the answer options could
be true? the options that cannot be true fail to
satisfy all of the conditions and are wrong. With
a variation of Which of the answer options
cannot be true? the options that must and could
be true fail to satisfy all of the conditions and are
wrong. With a variation of Which of the answer
options must be true? the options that could
and cannot be true fail to satisfy all of the
conditions and are wrong.
WHAT
WHAT
DISGUISES?
The LSAT uses four basic disguises for wronganswer options. Briefly stated, they are the
same-language disguise, the too-little-toomuch disguise, the true but disguise, and the
false-assertion disguise.
It is not beneath the LSAT to use disguises.
That is, a particular wrong-answer option may
simultaneously wear a same-language and a
false-assertion disguise.
The same-language disguise is simple. It
consists of a repetition of the same language that
was in the statement. This disguise attracts
test-takers because it connects directly to the
statement, and the language is usually so obscure
as to create an impression of subtle significance.
The too-little-too-much disguise is used to
overstate or understate something in the answer
option so that it fails to satisfy the conditions
required by the question.
The true but disguise means what it says.
The answer option is true, but it does not satisfy
the conditions required by the statement and the
question. It attracts the test-taker because it is
true and makes good sense. Its veracity obscures
its irrelevance.
The false-assertion disguise is used to
misstate information in the answer option. That
is, the answer option misinterprets or wrongly
characterizes a proposition or conclusion in the
problem. It is often combined with the samelanguage disguise. It is attractive because it is
obscure and authoritatively put.
ANSWER OPTIONS?
www.petersons.com
ARE WRONG-ANSWER
92
LSAT Success
This question calls for selection of the answer option that could satisfy the conditions set out in the
statement and question. Wrong-answer options
are those that cannot satisfy the conditions.
Answer option (A) is the best selection,
though it is unlikely that you would select it as
best after a quick reading. A careful reading will
disclose that there is nothing in the answer
option that conflicts with the conditions in the
statement and question. It is an attractive
cream-of-the-crop answer.
Answer option (B) is a cannot wronganswer option in a same-language disguise.
Every word of this answer is taken from the
statement, which connects it to the statement
and makes it seem attractive. However, it cannot
satisfy the statement conditions because, among
other reasons, confidence cannot support the
conclusions of media experts.
Option (C) is a cannot wrong-answer
option in too-much and same-language
disguises. The answer option overstates by using
much of the language of the statement to declare
that what was one measure is actually critical
and that it encompasses not just youthful
political involvement but all human behavior.
By using what could be misperceived as paraphrase rather than overstatement, the answer
might attract a few test-takers.
Option (D) is a cannot wrong-answer
option in a true but disguise. The answer option
is a true statement, but it is irrelevant to the
question posed. The authors conclusion, not the
media experts, is what is sought.
Option (E) is a cannot wrong-answer option
in false-assertion and same-language disguises.
It misstates the information given. Among many
other things, there is no mention in the statement
of the degree of teenage political activity involved,
nor is it suggested that such activity is related causally to exposure to the news.
LSAT Success
WHAT
WRONG-ANSWER DISGUISES?
The ways the LSAT varies the four basic disguises
are often fuzzy, although they are predictable.
Their outlines are always identifiable, and they
frequently can assist your identification of the
basic disguise and wrong-answer pattern.
However, for this knowledge to work to your
advantage, you have to have or develop a certain
tolerance of imprecision.
93
www.petersons.com
SUCCESS SESSION 6
IT IS IMPORTANT THAT YOU NOT GET
CAUGHT UP IN TRYING TO IDENTIFY WRONGANSWER DISGUISES PRECISELY. YOUR SENSE
THAT THEY ARE WRONG IS SUFFICIENT.
Same-Language Variations
There are four variations on the same-language
wrong-answer disguise that the LSAT usually uses.
The first reaches an incorrect conclusion,
using the language of the statement.
The second contradicts the statement,
using the language of the statement.
The third refers to a topic or point not in
the statement, using the language of the
statement.
The fourth misinterprets points made in
the statement, using the language of the
statement.
STATEMENT
It is worth repeating that studies show that teenagers exposure to the news is one measure of
the extent of youthful political involvement in
this society. In addition, the patterns of exposure to the news that develop during the teens
continue throughout life. Even though we have
confidence in these observations, the conclusions drawn from them by many media experts
are simply not valid because so little is understood about the variables that are involved.
Too-Little-Too-Much Variations
There are four primary variations on the toolittle-too-much wrong-answer disguise used by
the LSAT.
One contradicts the statement, taking a
part of the statement for the whole.
Another agrees with the statement, taking
a part of the statement for the whole.
Another is too narrow, answering part of
the question but not all of it.
The last is too broad, answering all of the
question and much more.
QUESTION
1. Which of the following can be
inferred from the statement?
ANSWER CHOICES
(A) Youthful political involvement in this
society is extensive.
(B) The variables that are involved in
patterns of exposure to the news
support the conclusions drawn by
many media experts.
(C) Patterns of exposure to the news are
measured by observation.
(D) The conclusions of many media
experts about youthful political
involvement are based on littleunderstood variables.
www.petersons.com
94
LSAT Success
QUESTION
1. Which of the following best describes
the observations of exposure to the
news made in the above statement?
True-But Variations
There are five primary variations on the true but
wrong-answer disguise used by the LSAT. The
wrong-answer option is true, but
ANSWER CHOICES
(A) It can be comprehensively measured
by youthful political involvement.
(B) It can measure youthful political
involvement in all societies.
(C) It can measure youthful political
involvement.
(D) It measures political involvement in
most societies, and its patterns
explain both teenage and adult
behavior.
LSAT Success
95
www.petersons.com
SUCCESS SESSION 6
The third uses a false assertion to agree
with the statement.
The fourth is a logical response but does
not match the premises of the statement.
QUESTION
1. Which of the following best summarizes the observations about exposure
to the news made in the above
statement?
STATEMENT
It is worth repeating that studies show that
teenagers exposure to the news is one
measure of the extent of youthful political
involvement in this society. In addition, the
patterns of exposure to the news that
develop during the teens continue
throughout life. Even though we have
confidence in these observations, the
conclusions drawn from them by many
media experts are simply not valid
because so little is understood about the
variables that are involved.
ANSWER CHOICES
(A) Accurate studies do not always reach
accurate conclusions.
(B) Exposure to the news changes over a
lifetime in response to changes in the
way news is reported.
(C) Those reporting news should take
teenagers interests into account.
(D) Exposure to the news is a lifelong
experience.
(E) Patterns of exposure to the news are
important to media experts.
QUESTION
1. Which of the following best describes
the conclusions reached by the media
experts referred to in the above
statement?
ANSWER CHOICES
(A) Youthful political involvement is a
measure of teenagers exposure to
the news.
(B) Youthful political activity is the result
of increased exposure to the news.
(C) Youthful political involvement is
measured by media experts.
(D) The conclusions are not valid because
the observations upon which they are
based are not complete.
This question again requires that you select the
answer option that could satisfy the conditions
set out in the statement. Wrong-answer options
are those that cannot satisfy the conditions.
Answer option (A) is a cannot wronganswer option that misinterprets or negates a
premise of the statement (that exposure to the
news measures political involvement) by making
the false assertion that political involvement
measures exposure to the news.
Option (B) is a cannot wrong-answer
option that uses a false assertion that contradicts
the statement. It declares that there is a causal
relationship between political activity and
exposure to the news.
Option (C) is a cannot wrong-answer
option involving a false assertion that agrees with
the statement to the extent that it notes that
political involvement is being measured. It is
False-Assertion Variations
There are four variations on the false-assertion
wrong-answer disguise:
The first misinterprets or negates a
statement premise.
The second uses a false assertion to
contradict the statement.
www.petersons.com
96
LSAT Success
AND
LSAT Success
97
www.petersons.com
Success Session 7
RELATIONSHIPS IN DETAIL
RELATIONSHIPS
PROBLEMS
CRITICAL DETAILS
There is one Relationships section-test on each
LSAT, which translates into some 25 percent of
the questions. Relationships question sets appear
in four forms: the line-up, cluster, map, and
schedule contexts.
Since there are four sets of Relationships
questions on each 35-minute section-test, this
might suggest that each of these contexts will be
represented on each section-test. In fact, a
section-test may include only two or three of
them. The line-up, cluster, and schedule contexts
appear with much greater frequency than does
the map context. In fact, the map context has
been almost completely phased out by the LSAT,
and it is unlikely that you will encounter it.
However, it will be covered just in case.
IN
WHAT
RELATIONSHIPS
WITH THE
CONSIDER
PROBLEMS?
99
SUCCESS SESSION 7
CHECK YOUR WORK TO BE CERTAIN THAT
EACH VARIABLE IS INCLUDED.
(A)
(B)
(C)
(D)
(E)
This is a must question. Refer to the visualization, and start work with answer option (A). You
determine that Jumbo weighs less than Giant,
which weighs less than Large. This means that
Large is heavier than Jumbo, and option (A) is
the best answer. Since you know there can be
only one answer that satisfies all conditions, and
you have determined that option (A) does this,
there is no reason to go any further in considering options. Testing every answer option can be
a big time-waster, avoid it.
www.petersons.com
1
2
3
4
5
0
1
2
3
4
100
LSAT Success
RELATIONSHIPS IN DETAIL
LSAT Success
101
www.petersons.com
SUCCESS SESSION 7
WHAT
This is a must question that involves a supplemental statement. The wrong answers are
those that could or cannot be true. First you
must add the supplemental information to the
visualization (K on page 101). By referring to the
augmented visualization, you determine that
Colossal and Large are the two heaviest sizes;
what is uncertain is the third size that is heavier
than Kennel. A review of the conditions discloses
that you have no information through which
certainty can be reached. Thus, it is likely that
the best answer will not involve the uncertainty.
Answer option (A) directly involves the uncertainty: Since Giant could be heavier than Kennel,
option (A) is wrong. Because Large weighs more
than all sizes other than Colossal, it must be
heavier than Kennel, making option (B) the best
answer. Since you know that only one answer
option satisfies all conditions, and you have
determined that (B) satisfies all conditions, there
is no reason to consider further options.
HOW
Kennel
Kennel
Kennel
Kennel
Kennel
is
is
is
is
is
This is a cannot question that involves a supplemental statement. The wrong answers are those
that must or could be true. First you must add the
supplemental information to the visualization. The
supplemental information, though conflicting with
that in question 5, yields the same visualization.
What is uncertain is the third size that is lighter
than Kennel. Again, a review of the conditions
discloses that you have no information through
which certainty can be reached.
Thus, once more it is likely that the best
answer will not involve the uncertainty. Answer
options (A) and (C) directly involve the uncertainty: since Kennel could be heavier than Small
and Giant, (A) and (C) are wrong. Because
Jumbo weighs less than all sizes other than
Medium, it must be lighter than Kennel, and
option (B) must be true, making it wrong when
what is asked for is a cannot answer. And
because Large weighs more than all sizes other
than Colossal, it must be heavier than Kennel,
and option (D) must be true, making it similarly
wrong. Thus, the best answer is (E).
www.petersons.com
CLUSTER CONTEXT?
102
LSAT Success
RELATIONSHIPS IN DETAIL
second one depicts the facts and rules if there is
just one director (Z) satisfying the conditions.
After completing the visualization, consider
the questions, recalling that you will be referring
only to the visualization as you work through the
questions.
This is a could question. You note from the visualizations that at least one member of the Audit
Committee is a member of the Planning Committee. Therefore, answer choice (A) cannot be true,
and it fails to meet the question conditions and is a
wrong-answer option. Next you consider answer
choice (B). From the visualizations you determine
that at least one member of the Audit Committee
is a member of the Finance Committee. Therefore,
answer choice (B) cannot be true, and it fails to
meet the question conditions and is a wrong-answer option. Then you consider answer choice
(C). The visualization shows you that at least one
This is a must question. Note from the visualizations that at least one member of the Finance
Committee is a member of the Planning Committee. Since this option satisfies the conditions of
the question, circle answer choice (A). Since you
know that only one answer option satisfies all
conditions, and you have determined that (A)
LSAT Success
103
www.petersons.com
SUCCESS SESSION 7
This is a must question with a supplemental
statement that establishes the second visualization as the accurate representation of the
questions conditions. Note on the visualization
that Gates (Z) is a member of the Audit and
Finance committees and a member of the
Planning and Shareholder Relations committees.
Since option (E) is the only one that satisfies the
conditions of the question, it is the best answer
choice. Also notice that this question is a
variation of the previous question 3. The LSAT
often uses this technique with Relationships
questions. Be on the alert for it!
www.petersons.com
104
LSAT Success
RELATIONSHIPS IN DETAIL
HOW
CONTEXT?
As with the other contexts, you begin by reading
the statement, with its facts and rules.
There are nine gasoline stations in the town
of Mountain View: Amoco, Boron, Chevron,
Exxon, Gulf, Hess, Mobil, Shell, and Texaco. The
stations are located as follows:
WHAT
LSAT Success
105
www.petersons.com
SUCCESS SESSION 7
3. Which of the following could be true?
due east
southeast
due north
northeast
southwest
0
1
2
3
4
www.petersons.com
1
2
3
4
5
106
LSAT Success
RELATIONSHIPS IN DETAIL
to be south of B, but they might not be. S and T do
not satisfy the conditions. Answer options (B), (C),
(D), and (E) cannot be true; none of them meets
the question conditions. Answer option (A) does
satisfy the conditions of the question, and it is the
best-answer option.
of A, and answer options (A), (B), (C), and (E) cannot be true. They do not meet the conditions and
are dumped. Option (D) does satisfy the conditions and is the best-answer option.
WHAT
HOW
4
5
6
7
8
This is a could question. Studying the visualization, you determine that A is due north of B and,
consequently, that B cannot be southeast of A.
Since cannot is the only wrong answer to a could
question, B does not satisfy the conditions of the
question. There is no information in the statement
or question from which you can determine that B
and A are not westernmost among the stations,
nor can you determine how far north of B station
A is located. So if A is located sufficiently far north,
all stations other than B could be southeast of A.
This means that seven stations could be southeast
LSAT Success
SCHEDULE CONTEXT?
107
www.petersons.com
SUCCESS SESSION 7
The many facts and rules tell you that this is the
schedule context and suggest that order and
location are the ideas on which the question set
will focus. A brief survey of the questions
confirms this suggestion. Now you are ready to
visualize a schedule.
The facts tell you that there are twelve acts
(individuals), four time slots, and three rings
(locations). The individuals are A, B, C, D, E, H,
J, L, S, T, U, and W. The intersection of time and
location determines the nodes of your visualization, which results in a 3 x 4 matrix. Check to
be certain that each variable is included, and
apply the rules to the individuals, noting the
results in the matrix as follows.
The first rule tells you that T, U, and W
perform only in ring 2. The second rule tells you
that L and B perform only in ring 3. Since rule
three provides that animal acts cannot be
presented in adjacent rings at the same time, rule
four provides that dogs and lions (and acrobats)
perform at the same time, and rule two provides
that lions perform in ring 3, it follows that dogs
must perform in ring 1 (not adjacent to another
animal act) and acrobats in ring 2. You have
now determined all of the acts for ring 2: A, T,
U, and W.
Rule two provides that C, E, and H cannot
perform in ring 3, and ring 2 is full, so C, E, and
H must perform in ring 1 with D. This determines all of the acts for ring 1: C, D, E, and H.
www.petersons.com
108
LSAT Success
RELATIONSHIPS IN DETAIL
5. If the acrobats are presented in the first
segment, which of the following CANNOT
be true?
question, and is the best answer. There is no reason to consider option (E).
3. If the lions are presented in the first
segment, which of the following must be
true?
(A)
(B)
(C)
(D)
(E)
(E)
LSAT Success
(A)
(B)
(C)
(D)
The
The
The
The
The
A practice exercise
To get a sense of each of the types of Relationships questions, turn to one of the complete
tests in the book. Scan through each of the
Relationships sets and identify the type of
context that each question set presents.
109
www.petersons.com
Success Session 8
ARGUMENTS PROBLEMS IN DETAIL
below. In this way, you create four sets of Arguments questions in each 35-minute section-test.
IN
Question
Question
Question
Question
set
set
set
set
1:
2:
3:
4:
16
712
1318
1924+
ARGUMENTS
CONSIDERED?
QUESTION SETS BE
questions
questions
questions
questions
111
SUCCESS SESSION 8
HOW
first two sets of questions and guessing your answers on the last two sets or omitting twelve questions as you work through the four question sets,
passing over twelve long question-and-answer
combinations or confusing questions.
HOW
ARGUMENTS
CONSTRUCTED?
ARE
WORK?
ARGUMENTS
QUESTIONS
PROBLEMS
The Directions
www.petersons.com
DO
112
LSAT Success
The Question
tions when working through Arguments questionsthis seems to pose no great challenge.
And it is not much of a challenge if you
heed the directions we looked at earlier.
Translated, the LSATs directions tell you that
unjustified assumptions are those that exceed the
terms of the statement, are inconsistent with the
terms of the statement, or are unfounded given
the terms of the statement. Avoid these things,
and you will identify wrong-answer options
readily and efficiently select best answers, right?
Not exactly!
Unfortunately, experience shows that this is
easier said than done. Assumptions are the
stock-in-trade of the able student, for whom the
ability to justify assumptions and positions
persuasively is a necessary skill rewarded in
examination papers, essays, debates, and other
such exercises. The temptation to apply this
ability in the context of the LSAT can be almost
irresistible to the able student. When a test-taker
gives in to this temptation, the result is the
erroneous selection of a wrong-answer option as
the best answer to an Arguments question, so
fight the unjustified-assumption trap with
everything youve got. Your Arguments sectiontest performance will benefit.
The Statement
Arguments statements are generally short. Their
brevity can make them seem deceptively simple.
The brevity is often the result of the LSATs
strategy of obfuscation by omission. Many
Arguments questions seek omitted evidence or a
conclusionin other words, the information
required to complete the argument.
Arguments statements always involve a
position or point of view. Although the position
or point of view may not be clearly articulated,
the statement is never neutral. Identify the point
of view presented. It makes dealing with the
problem much more straightforward.
A statement consists of 75 to 125 words. It
defines the context for either one or two question-and-answer sets. Usually a statement supporting two questions has a relatively smaller reading
burden than a statement supporting only one.
LSAT Success
The Answers
The answer options complete the Arguments
question; without them the question has little
practical meaning. Take a typical Arguments
question: Which of the following is the underlying point of the above? You are being asked,
Whats the point? As you know, this is not an
invitation to extemporize or demonstrate your
ability to synthesize. Your activity should be
confined to the selection of the answer options
that best satisfy the conditions of the question
and statement. You achieve this by viewing each
answer option in relation to the question. Only
then can you select answers in the way the LSAT
requires.
113
www.petersons.com
SUCCESS SESSION 8
THE ANSWER OPTIONS PRESENT YOU WITH
A FIELD OF CHOICES FROM WHICH YOU
MUST DECIDE. YOU SELECT ONLY FROM
THE FIVE OPTIONS PRESENTED.
same-language disguise
too-little-too-much disguise
true but disguise
false-assertion disguise
the
the
the
the
same-language disguise
too-little-too-much disguise
true but disguise
false-assertion disguise
the
the
the
the
same-language disguise
too-little-too-much disguise
true but disguise
false-assertion disguise
WHAT
www.petersons.com
the
the
the
the
TECHNIQUE FOR
QUESTIONS?
ARGUMENTS
114
LSAT Success
able to tell as quickly as possible which information in the statement is relevant to the question
and which is superfluous. Reading the question
first gives you clues about what will be relevant.
Now read the statement. As you do so,
highlight or underline key words, assertions, and
the conclusion, if there is one. If there is no
conclusion in the statement, it is likely that the
question will ask for a conclusion in some form.
Next, work through each answer option. As
you work through the answer options, you will
use comparison and contrast to identify the
options that fit a wrong-answer pattern, keeping
alert for disguises. When you find a condition in
an answer option that fails to satisfy the requirements of the question or statement, you chuck it.
Repeating this process as necessary, you select the
best answer option and circle it in the test book.
Once you have completed all of the
questions in the set of six, you transfer all your
answers to the answer sheet at the same time.
This gives you a natural, albeit brief, break before
you move on to the next Arguments questionanswer set.
HOW
BASIC
ARGUMENTS
QUESTIONS?
The LSAT uses six basic question types in the Arguments section-test. The techniques used to
work through them are detailed in the following
paragraphs. These techniques are applicable to
any version or variation of the six questions. It
might also help you familiarize yourself with the
thinking required if, after this discussion, you review each question and answer option, specifically identifying the disguises used by the LSAT.
(For example, option (C) in question 4 uses the
same language as the statement to disguise the
wrong answer. Option (B) in question 3 takes advantage of the true but disguise, and option (B) in
question 1 applies the false-assertion disguise to
a wrong-answer choice.)
Description Questions
Description questions make direct reference to
the argument. There is no need to go beyond the
argument to identify the answer option that
satisfies the conditions of the question and
statement. Description questions take two forms.
The first asks you to identify a point, purpose,
premise, or form of reasoning used in the
argument. The second asks you to identify a
stated presumption.
LSAT Success
115
www.petersons.com
SUCCESS SESSION 8
principle in common with or analogous to that
of the statement. Rarely, you might be asked to
identify a logical flaw in a statement. It takes the
form The above is most like which of the
following?
The question in this example requires careful reading. It asks you what Harry presumes Mark meant.
Answer options (A) and (D) cannot satisfy the required conditions because they compare such
items as the likelihood of battle fatigue and differences in job stress. Nothing in the statement suggests such comparisons. Option (C) cannot satisfy
the required conditions because there is nothing
in the statement to suggest a disagreement about
the nature of battle fatigue. Option (E) cannot satisfy the required conditions because it is Harrys
position, not Harrys interpretation of Marks position. Option (B) is the best selection because it
does not conflict with the conditions of the statement in any way.
Extension Questions
Extension questions require you to go beyond
the argument to identify the answer option that
satisfies the question and statement conditions.
You will have to draw inferences or define
assumptions or reach conclusions. The answer
options do not connect to the statement directly.
Extension questions take four forms, in
which you are asked to identify a matching
pattern, determine inferences and conclusions,
recognize assumptions required for a conclusion,
or assess evidence.
Pattern questions ask you directly to select
the answer option that has a structure or
www.petersons.com
116
LSAT Success
While the pattern is not always based on sentence structure, very often an analysis at the sentence level will quickly produce the best answer.
Inference and conclusion questions ask you
to select the answer option that expresses an
inference or conclusion drawn from the statement. The format is very direct, asking, Which
of the following conclusions can be logically
inferred?
LSAT Success
117
www.petersons.com
SUCCESS SESSION 8
6. The British economy is stagnant, which
can only be the result of its overgenerous
governmental programs of social aid.
Arguments statements and answer options are replete with cue words. These words indicate the
nature of the information that follows them. For
example, the appearance of the word therefore
cues you that the conclusion is coming up. There
are a bundle of similar cue words. They are not
worth committing to memory, but a read-through
may prove beneficial to your test performance.
Some conclusion cues follow.
This evidence-assessment question asks you to select the answer option that could weaken the statement. The fact that the word most is used to
modify weaken is of no importance; this degree
of refinement is virtually never involved on the
LSAT. You will need to identify the conclusion in
the statementThe British economy is stagnant.
Answer option (A) does not satisfy the
requirements of the question and statement. It is
wrong because nothing in the option suggests
the proportion of income the Germans spend on
social-aid programs. Also, it involves only medical
care, which is just one social-aid program;
therefore, it cannot be related to the statement.
Options (B) and (C) cannot satisfy the statement
conditions and are wrong because, among other
things, they fail to indicate the condition
(stagnant?) of the economies to which Britains
economy is being compared. Option (E) is wrong
because it fails to indicate whether the amount
Italy spends on its few social-aid programs is
more, less, or the same as that spent by Britain.
It cannot be related to the statement. This leaves
(D) as the answer option that weakens the
www.petersons.com
so
indicates
therefore
suggests
hence
proves
thus
means
accordingly
shows
can be inferred
nevertheless
results
however
follows
118
given
inasmuch as
since
as
because
but
assume
except
suppose
despite
if
notwithstanding
insofar as
although
LSAT Success
Success Session 9
PASSAGES PROBLEMS IN DETAIL
OF
HOW
PASSAGES
CONSTRUCTED?
ARE
PROBLEMS
For the most part, Passages are taken from law reviews and similar journals. This is not to suggest that
some knowledge of law is required or even beneficial in working through Passages problems. In fact,
the statements are chosen and edited to avoid jargon and legalese. They involve diverse topicsfrom
technical subjects to topics drawn from the social
sciences, the humanities, and other fields.
PASSAGES STATEMENTS ARE CHOSEN FOR
THEIR COMMON CHARACTERISTICS, NOT
FOR THEIR SUBJECT MATTER.
ALL OF THE
point of purpose
key words or concepts
authorities
enumerations
unusual words or phrases
competing perspectives
Line
119
SUCCESS SESSION 9
5
10
15
20
25
30
35
40
45
50
55
www.petersons.com
60
65
70
75
80
CONSIDER
120
LSAT Success
The Statement
Passages statements obfuscate by their basic
structure. They are relatively long, generally
between 500 and 550 words, and much of the
material in them is not relevant to the questions.
But the length of the statement presents most
readers with fewer difficulties than do its other
attributes. The absence of familiar structure can
be disconcerting, and there are no titles to go by.
Each statement has been heavily edited beforehand, with the result that the purpose of the
statement is unclear, especially since it appears
outside of its original context. The statement is
compressed into three to five paragraphs, and
the paragraphs often do not reflect different
LSAT Success
The Question
Passages questions are limited in their coverage
and structure. They directly state the decision to
be made by test-takers. They do not contain
supplemental statements. Passages questions do
not modify or add to the statement, and each
question is independent of every other one.
121
www.petersons.com
SUCCESS SESSION 9
answer option that must, could, or cannot satisfy
the question and statement conditions. This
means that, before you work through the answer
options, you must first determine the nature and,
to the degree possible, the focus of the question.
Is it must, could, or cannot? Does it focus on an
enumeration, authority, unusual word, or some
other statement characteristic? By doing this, you
can take best advantage of the time available,
minimizing the number of times you deal with
the statement and answer options.
identify a perspective
identify a context
identify a pattern
assess evidence
The Answers
The answer options complete the question,
which, by itself, means very little. The typical
Passages question asks, Which of the following
best summarizes the point? It does no more
than tell you that the selection of an answer
option will be necessary. Only when it is
connected to the answer options does the
question become sufficiently specific to permit
answer selection.
THE ANSWER OPTIONS PRESENT THE REAL
DECISION TO YOU.
As with Arguments, you will use comparison
and contrast to work through the answer
options. These techniques assist you in detecting
wrong-answer disguises and identifying differences between options.
With most Passages problems, it is not clear
whether the question requires the selection of an
www.petersons.com
122
LSAT Success
the
the
the
the
same-language disguise
too-little-too-much disguise
true but disguise
false-assertion disguise
the
the
the
the
same-language disguise
too-little-too-much disguise
true but disguise
false-assertion disguise
the
the
the
the
same-language disguise
too-little-too-much disguise
true but disguise
false-assertion disguise
LSAT Success
123
www.petersons.com
SUCCESS SESSION 9
10
15
20
25
30
35
40
45
50
55
60
www.petersons.com
124
LSAT Success
HOW
Description Questions
Description questions always refer directly to
the passage statement. Refer to the statement if
LSAT Success
125
www.petersons.com
SUCCESS SESSION 9
Specific detail questions also have a direct
format. They focus on enumerations, a key word
or concept, or an unusual word or phrase.
Specific detail questions usually require a must
or cannot answer option.
the
the
the
the
the
blackmailer
offenders
legislature
criminal
bribed officials
Extension Questions
Extension questions always require that you go
beyond the statement to arrive at the answer.
You are required to make inferences and
assumptions and draw conclusions from material
you find in the statement. Reference to the
statement may be helpful.
Extension questions take four forms. You
are asked to determine a perspective, select the
context from which the statement came, identify
a pattern, or assess evidence.
Perspective questions ask you directly to
select the answer option that reflects the view,
attitude, or purpose of the statements author or
some other person involved with the statement.
They may seek either general or specific perspectives. While they may go beyond the statement,
they also may connect directly to material in the
statement, often bridging the area between
description and extension questions.
www.petersons.com
126
LSAT Success
LSAT Success
127
www.petersons.com
SUCCESS SESSION 9
7. Which of the following, if true, most
strengthens the authors position on the
legality of blackmail?
Once again, this is a could question. If answer choice (A) were true, it would weaken the
authors position, not strengthen it. It cannot satisfy the question and statement conditions and is
wrong. Answer options (C), (D), and (E) do not
satisfy the required conditions, since they refer to
matters not included in the statement. Use of the
contrast technique makes it clear that there is no
practical difference between answer options (C)
and (D), and, consequently, each is wrong. This
leaves answer option (B) as the best answer.
The techniques used to work through the
seven examples of Passages questions are applicable to all versions of those questions. If you
found yourself attracted to a wrong-answer option
as you worked through the questions, you might
benefit from returning to that option and identifying the disguise that made it attractive. By becoming familiar with the disguises that attract you, you
can avoid being misled by them in the future.
www.petersons.com
128
LSAT Success
Success Session 10
RELATIONSHIPS PROBLEMS AND THE 9-12-18 SYSTEM
HOW
DOES THE
APPLY TO YOUR
TRAINING?
9-12-18 SYSTEM
RELATIONSHIPS
HOW
DO YOU USE
SECTION-TESTS?
RELATIONSHIPS
129
SUCCESS SESSION 10
the answers you selected during this 3-minute
period. If you did not change an answer, simply
place a box beside the circled answer. If you did,
place the box beside the new selection. The
boxed answers are your 12-minute set.
In the final 6 minutes, go back and answer
any questions you did not get to during the first
12 minutes, or review and change answers if you
wish. Be flexible. Concentrate on questions
about which you are uncertain, and take care not
to rethink previously selected answers about
which you are confident. Place an asterisk
beside each of the answers you selected during
this 6-minute period, without erasing or changing
the circles and boxes you made during the first
12 minutes. If you did not change an answer,
place an asterisk beside the boxed answer. If you
did, place an asterisk beside the new selection.
The answers identified with asterisks are your
12-minute set.
After you have completed the pretest, turn
to the Answer Keys for the question-answer set
you completed at the back of the book. For each
set, determine the number of best answers you
selected during the 9-minute set (circled),
12-minute set (boxed), and 18-minute set
(asterisked). Apply the numbers to the following
formula, and complete the calculations indicated.
The largest total indicates the optimal
number of question sets (two, three, or four)
that you transfer to the Pretest Performance
Tracking Worksheet (page 75). As you develop
this record, it will clearly indicate the number of
Relationships question sets you should plan to
complete when taking the LSAT. Completing this
number will yield the highest test score for you.
record. You should work through the Relationships training techniques at least six times and
complete at least six question-answer sets.
Working on more question-answer sets could
result in continued improvement. If it does, keep
going. If you become consistent in your performance, stop.
Upon completion of at least six training and
update sessions, your performance record is
likely to have stabilized, and your plan for the
Relationships section-test will be set. Your plan
will indicate the number of question-answer sets
you should complete and the number you should
guess on the Relationships section-test at any
sitting of the LSAT.
HOW
RELATIONSHIPS
SETS?
DO YOU TAKE
QUESTION-ANSWER
www.petersons.com
Relationships Set
Circles ______________ 5 + 0 = ________
Boxes _______________ 4 + 1 = ________
Asterisks_____________ 3 + 2 = ________
Largest total ________
To repeat, every time you work quickly
through Sessions 5 and 7, reviewing the main
points, you complete one of the question-answer
sets. This process will take about 30 minutes.
Dont be tempted to complete a set without
working through the techniques; it is familiarity
with and proficiency in the use of the techniques
that will have the greatest impact on your
performance. Conversely, working through a
series of question-answer sets without first
working through the techniques is the least
130
LSAT Success
WHAT
SHOULD
SETS?
ANSWER SETS?
Once you have completed six training workthroughs, you probably will have realized the highimpact gain that you were looking for before taking the LSAT. For most, further workthroughs and
sets will have much less impact, if any. But it is
important to maintain familiarity and proficiency,
and some people find they can also maintain control through regular practice. In any event, you
LSAT Success
131
www.petersons.com
Success Session 11
ARGUMENTS PROBLEMS AND THE 9-12-18 SYSTEM
HOW
9-12-18
PROBLEMS?
SYSTEM TO
ARGUMENTS
Even after you have arrived at a stable balance between speed and accuracy, working through Arguments question-answer sets will maintain and enhance your level of familiarity and proficiency.
Always work through the technique sessions first,
and then monitor your performance with 9-12-18
question-answer sets that appear in the simulation
LSATs that appear in this book.
HOW
DO YOU USE
QUESTION-ANSWER
ARGUMENTS
SETS?
133
SUCCESS SESSION 11
UPON COMPLETION OF THE SIX TRAINING
AND UPDATE SEQUENCES, YOU SET YOUR
PLAN FOR THE ARGUMENTS SECTION-TESTS.
HOW
ARGUMENTS
SETS?
DO YOU TAKE
QUESTION-ANSWER
www.petersons.com
134
LSAT Success
SHOULD
WHAT
SETS?
ANSWER SETS?
LSAT Success
135
www.petersons.com
Success Session 12
PASSAGES PROBLEMS AND THE 9-12-18 SYSTEM
HOW
9-12-18
PASSAGES TRAINING?
ing three of the four question sets on a sectiontest (and guessing on the fourth set). Working
with the Passages training techniques you have
learned, you will use the 9-12-18 process to
update your performance record with the results.
After you have worked through a number of
question-answer sets, your performance record
may suggest making a change in your plan for
optimal performance. For instance, your efficiency may have improved so that your performance record shows you will achieve a better
score by completing all four sets of questions on
the Passages section-test rather than by completing only three question-answer sets and guessing
on one. This calls for a change in your test plan
for the section-test.
SYSTEM TO YOUR
HOW
DO YOU USE
QUESTION-ANSWER
PASSAGES
SETS?
YOUR
PLAN?
137
SUCCESS SESSION 12
HOW
PASSAGES
SETS?
DO YOU TAKE
QUESTION-ANSWER
www.petersons.com
138
LSAT Success
HOW
LSAT Success
WHAT
139
www.petersons.com
Success Session 13
THE WRITING SAMPLE
THERE IS NO EVIDENCE THAT LAW SCHOOLS
USE THE WRITING SAMPLE TO IDENTIFY
APPLICANTS WITH GREAT POTENTIAL AS
LAW STUDENTS OR LEGAL WRITERS.
WHAT
LSATS OBJECTIVES
WRITING SAMPLE?
ARE THE
FOR THE
FOR THE
USE YOUR
ON THE
141
SUCCESS SESSION 13
AVOID PRODUCING A WRITING SAMPLE
THAT IS HARD TO READ.
It is most important that you avoid the temptation to extemporize or be inventive. This can
best be ensured by sticking to the directions.
OF THE
The Directions
The Writing Sample directions tell you that how
well you write is more important than how
much you write. Law schools are mainly interested in how clearly you express the position
you take on the topic; how carefully you support
that position; and your organization, vocabulary,
and writing mechanics.
You are told to write on the assigned topic
only. Under no circumstances should you write
on a topic of your choosing. Why prepare a
writing sample in advance on your own topic
when you can use the LSATs topic and satisfy
the expectations exactly? No special knowledge
about the topic is expected or required, but you
are expected to write on the topic assigned.
You are also advised that there is no right
or wrong answer to the topic. The LSAT does
not evaluate the position you take, nor are the
law schools provided with model answers or
model Writing Samples. As with all LSAT
questions, the LSAT uses the Writing Sample
topic to elicit a response. There is no interest in
your knowledge of the topic.
Time is short. You are told that you have
30 minutes to plan, organize, and write your
sample. This places a premium on your plan for
completing the Writing Sample. Remember
coloring books and being told to color inside the
lines? The Writing Sample is like that: Only the
writing on the lined area in the booklet is
reproduced and sent to the law schools. The
space provided is more than enough to accommodate your sample, provided you follow your
plan. Just be sure that you limit your writing to a
reasonable size that fits easily between the lines.
Use the black pen provided by the LSAT so that
your Writing Sample will reproduce clearly.
Legible handwriting is imperative. If your
writing is not entirely legible, print. If your printing is not entirely legible, draw. Make certain that
your writing sample can be read easily.
www.petersons.com
The Topic
All Writing Sample topics have a similar, predictable layout. They are made up of three componentspositions, conditions, and evidence. In
the following example, the components are
spelled out. These headings do not appear on the
LSAT, but the components are obvious and
cannot be confused.
There are two positions and two conditions
in each topic. The positions are stated first, the
conditions follow, and then comes the evidence.
Positions
As a trustee of Whistler College, you will
soon vote on how a recent donation of
$1 million will be used. Write an argument
in support of applying the donation to the
development of either the Art Department
or the Economics Department. Two considerations guide your decision:
Conditions
Enrollments at Whistler have been
decreasing in the last few years.
Disharmony between faculty and
students has been increasing in the last
few years.
Evidence
For years, the Art Department has had an
inadequate staff and limited course
offerings, and what has been offered has
not been of good quality. This gift would
enable the college to hire four excellent
people, including an artist-in-residence.
One or two could round out the course
offerings in art history, and others could
do something about the lack of courses in
studio art. That should change the
departments image. It should also attract
some new art majors or at least keep
142
LSAT Success
LSAT Success
143
www.petersons.com
SUCCESS SESSION 13
Faculty-student disharmony is the second
condition to be addressed. Evidence item 3 is
drawn directly from the topic, while item 4 is an
extension of material given in the topic.
YOUR
www.petersons.com
144
LSAT Success
1234567890123456789012345678901
123456789012345678901234567890 1
123456789012345678901234567890 1
123456789012345678901234567890 1
123456789012345678901234567890 1
123456789012345678901234567890 1
123456789012345678901234567890 1
123456789012345678901234567890 1
123456789012345678901234567890 1
123456789012345678901234567890 1
123456789012345678901234567890 1
123456789012345678901234567890 1
123456789012345678901234567890 1
123456789012345678901234567890 1
123456789012345678901234567890 1
123456789012345678901234567890 1
123456789012345678901234567890 1
123456789012345678901234567890 1
123456789012345678901234567890 1
123456789012345678901234567890 1
123456789012345678901234567890 1
123456789012345678901234567890 1
123456789012345678901234567890 1
123456789012345678901234567890 1
123456789012345678901234567890 1
123456789012345678901234567890 1
123456789012345678901234567890 1
123456789012345678901234567890 1
123456789012345678901234567890 1
123456789012345678901234567890 1
123456789012345678901234567890 1
123456789012345678901234567890 1
123456789012345678901234567890 1
123456789012345678901234567890 1
123456789012345678901234567890 1
123456789012345678901234567890 1
123456789012345678901234567890 1
123456789012345678901234567890 1
123456789012345678901234567890 1
123456789012345678901234567890 1
123456789012345678901234567890 1
123456789012345678901234567890 1
123456789012345678901234567890 1
123456789012345678901234567890 1
123456789012345678901234567890 1
123456789012345678901234567890 1
123456789012345678901234567890 1
123456789012345678901234567890 1
123456789012345678901234567890 1
123456789012345678901234567890 1
123456789012345678901234567890 1
123456789012345678901234567890 1
123456789012345678901234567890 1
123456789012345678901234567890 1
1234567890123456789012345678901
WHAT
LSAT Success
PRINCIPLES OF VOCABULARY,
WRITING SAMPLE
PERFORMANCE?
145
www.petersons.com
SUCCESS SESSION 13
pronoun other than I. It, they, and the like
often have unclear referents. Avoid legalese in
any form.
You should already be familiar with the
rules of grammar required to produce a Writing
Sample. The following alerts are reminders of a
few matters of particular importance: Sentences
must be complete; adjectives modify nouns and
adverbs modify verbs or adjectives; parallel
construction is required; and double negatives
should be avoided.
www.petersons.com
146
LSAT Success
Success Session 14
WRITING SAMPLES
WRITING
PRACTICE SESSIONS
HOW
YOU COMPLETE?
Complete one to five writing samples. Once you
complete a Writing Sample that satisfies your
plan, there is no improvement to be realized by
completing additional samples. You can use any
remaining sample topics to maintain your
familiarity with the Writing Sample techniques.
While there is little reason to seek out
further Writing Samples, the disclosed tests
offered by Law School Admission Services
include a Writing Sample topic. Some LSAT
preparation books also provide topics, but if you
use such preparation books, you must make
certain that the topics are constructed to the
same specifications as those of the LSAT.
FOR THE
147
SUCCESS SESSION 14
be writing under pressure and are limited to a
short time in which to produce the finished
sample.
Your writing should be confined to the
lined area in the booklet, since only this area will
www.petersons.com
148
LSAT Success
WRITING SAMPLES
123456789012345678901234567890121234567890123456789012345678901212
2345678901234567890123456789012123456789012345678901234567890121 2
1
2345678901234567890123456789012123456789012345678901234567890121 2
1
2345678901234567890123456789012123456789012345678901234567890121 2
1
2345678901234567890123456789012123456789012345678901234567890121 2
1
2345678901234567890123456789012123456789012345678901234567890121 2
1
2345678901234567890123456789012123456789012345678901234567890121 2
1
2345678901234567890123456789012123456789012345678901234567890121 2
1
2345678901234567890123456789012123456789012345678901234567890121 2
1
2345678901234567890123456789012123456789012345678901234567890121 2
1
2345678901234567890123456789012123456789012345678901234567890121 2
1
2345678901234567890123456789012123456789012345678901234567890121 2
1
2345678901234567890123456789012123456789012345678901234567890121 2
1
2345678901234567890123456789012123456789012345678901234567890121 2
1
2345678901234567890123456789012123456789012345678901234567890121 2
1
2345678901234567890123456789012123456789012345678901234567890121 2
1
2345678901234567890123456789012123456789012345678901234567890121 2
1
2345678901234567890123456789012123456789012345678901234567890121 2
1
2345678901234567890123456789012123456789012345678901234567890121 2
1
2345678901234567890123456789012123456789012345678901234567890121 2
1
2345678901234567890123456789012123456789012345678901234567890121 2
1
2345678901234567890123456789012123456789012345678901234567890121 2
1
2345678901234567890123456789012123456789012345678901234567890121 2
1
2345678901234567890123456789012123456789012345678901234567890121 2
1
2345678901234567890123456789012123456789012345678901234567890121 2
1
2345678901234567890123456789012123456789012345678901234567890121 2
1
2345678901234567890123456789012123456789012345678901234567890121 2
1
2345678901234567890123456789012123456789012345678901234567890121 2
1
2345678901234567890123456789012123456789012345678901234567890121 2
1
2345678901234567890123456789012123456789012345678901234567890121 2
1
2345678901234567890123456789012123456789012345678901234567890121 2
1
2345678901234567890123456789012123456789012345678901234567890121 2
1
2345678901234567890123456789012123456789012345678901234567890121 2
1
2345678901234567890123456789012123456789012345678901234567890121 2
1
2345678901234567890123456789012123456789012345678901234567890121 2
1
2345678901234567890123456789012123456789012345678901234567890121 2
1
2345678901234567890123456789012123456789012345678901234567890121 2
1
2345678901234567890123456789012123456789012345678901234567890121 2
1
2345678901234567890123456789012123456789012345678901234567890121 2
1
2345678901234567890123456789012123456789012345678901234567890121 2
1
2345678901234567890123456789012123456789012345678901234567890121 2
1
2345678901234567890123456789012123456789012345678901234567890121 2
1
2345678901234567890123456789012123456789012345678901234567890121 2
1
12345678901234567890123456789012123456789012345678901234567890121 2
123456789012345678901234567890121234567890123456789012345678901212
LSAT Success
149
www.petersons.com
SUCCESS SESSION 14
123456789012345678901234567890121234567890123456789012345678901212
2345678901234567890123456789012123456789012345678901234567890121 2
1
2345678901234567890123456789012123456789012345678901234567890121 2
1
2345678901234567890123456789012123456789012345678901234567890121 2
1
2345678901234567890123456789012123456789012345678901234567890121 2
1
2345678901234567890123456789012123456789012345678901234567890121 2
1
2345678901234567890123456789012123456789012345678901234567890121 2
1
2345678901234567890123456789012123456789012345678901234567890121 2
1
2345678901234567890123456789012123456789012345678901234567890121 2
1
2345678901234567890123456789012123456789012345678901234567890121 2
1
2345678901234567890123456789012123456789012345678901234567890121 2
1
2345678901234567890123456789012123456789012345678901234567890121 2
1
2345678901234567890123456789012123456789012345678901234567890121 2
1
2345678901234567890123456789012123456789012345678901234567890121 2
1
2345678901234567890123456789012123456789012345678901234567890121 2
1
2345678901234567890123456789012123456789012345678901234567890121 2
1
2345678901234567890123456789012123456789012345678901234567890121 2
1
2345678901234567890123456789012123456789012345678901234567890121 2
1
2345678901234567890123456789012123456789012345678901234567890121 2
1
2345678901234567890123456789012123456789012345678901234567890121 2
1
2345678901234567890123456789012123456789012345678901234567890121 2
1
2345678901234567890123456789012123456789012345678901234567890121 2
1
2345678901234567890123456789012123456789012345678901234567890121 2
1
2345678901234567890123456789012123456789012345678901234567890121 2
1
2345678901234567890123456789012123456789012345678901234567890121 2
1
2345678901234567890123456789012123456789012345678901234567890121 2
1
2345678901234567890123456789012123456789012345678901234567890121 2
1
2345678901234567890123456789012123456789012345678901234567890121 2
1
2345678901234567890123456789012123456789012345678901234567890121 2
1
2345678901234567890123456789012123456789012345678901234567890121 2
1
2345678901234567890123456789012123456789012345678901234567890121 2
1
2345678901234567890123456789012123456789012345678901234567890121 2
1
2345678901234567890123456789012123456789012345678901234567890121 2
1
2345678901234567890123456789012123456789012345678901234567890121 2
1
2345678901234567890123456789012123456789012345678901234567890121 2
1
2345678901234567890123456789012123456789012345678901234567890121 2
1
2345678901234567890123456789012123456789012345678901234567890121 2
1
2345678901234567890123456789012123456789012345678901234567890121 2
1
2345678901234567890123456789012123456789012345678901234567890121 2
1
2345678901234567890123456789012123456789012345678901234567890121 2
1
2345678901234567890123456789012123456789012345678901234567890121 2
1
2345678901234567890123456789012123456789012345678901234567890121 2
1
2345678901234567890123456789012123456789012345678901234567890121 2
1
2345678901234567890123456789012123456789012345678901234567890121 2
1
2345678901234567890123456789012123456789012345678901234567890121 2
1
2345678901234567890123456789012123456789012345678901234567890121 2
1
2345678901234567890123456789012123456789012345678901234567890121 2
1
2345678901234567890123456789012123456789012345678901234567890121 2
1
2345678901234567890123456789012123456789012345678901234567890121 2
1
2345678901234567890123456789012123456789012345678901234567890121 2
1
2345678901234567890123456789012123456789012345678901234567890121 2
1
2345678901234567890123456789012123456789012345678901234567890121 2
1
2345678901234567890123456789012123456789012345678901234567890121 2
1
2345678901234567890123456789012123456789012345678901234567890121 2
1
2345678901234567890123456789012123456789012345678901234567890121 2
1
2345678901234567890123456789012123456789012345678901234567890121 2
1
2345678901234567890123456789012123456789012345678901234567890121 2
1
2345678901234567890123456789012123456789012345678901234567890121 2
1
2345678901234567890123456789012123456789012345678901234567890121 2
1
2345678901234567890123456789012123456789012345678901234567890121 2
1
2345678901234567890123456789012123456789012345678901234567890121 2
1
2345678901234567890123456789012123456789012345678901234567890121 2
1
2345678901234567890123456789012123456789012345678901234567890121 2
1
2345678901234567890123456789012123456789012345678901234567890121 2
1
2345678901234567890123456789012123456789012345678901234567890121 2
1
2345678901234567890123456789012123456789012345678901234567890121 2
1
2345678901234567890123456789012123456789012345678901234567890121 2
1
2345678901234567890123456789012123456789012345678901234567890121 2
1
2345678901234567890123456789012123456789012345678901234567890121 2
1
2345678901234567890123456789012123456789012345678901234567890121 2
1
2345678901234567890123456789012123456789012345678901234567890121 2
1
2345678901234567890123456789012123456789012345678901234567890121 2
1
2345678901234567890123456789012123456789012345678901234567890121 2
1
2345678901234567890123456789012123456789012345678901234567890121 2
1
2345678901234567890123456789012123456789012345678901234567890121 2
1
2345678901234567890123456789012123456789012345678901234567890121 2
1
2345678901234567890123456789012123456789012345678901234567890121 2
1
2345678901234567890123456789012123456789012345678901234567890121 2
1
2345678901234567890123456789012123456789012345678901234567890121 2
1
2345678901234567890123456789012123456789012345678901234567890121 2
1
2345678901234567890123456789012123456789012345678901234567890121 2
1
2345678901234567890123456789012123456789012345678901234567890121 2
1
2345678901234567890123456789012123456789012345678901234567890121 2
1
2345678901234567890123456789012123456789012345678901234567890121 2
1
2345678901234567890123456789012123456789012345678901234567890121 2
1
2345678901234567890123456789012123456789012345678901234567890121 2
1
2345678901234567890123456789012123456789012345678901234567890121 2
1
2345678901234567890123456789012123456789012345678901234567890121 2
1
2345678901234567890123456789012123456789012345678901234567890121 2
1
12345678901234567890123456789012123456789012345678901234567890121 2
123456789012345678901234567890121234567890123456789012345678901212
www.petersons.com
150
LSAT Success
WRITING SAMPLES
123456789012345678901234567890121234567890123456789012345678901212
2345678901234567890123456789012123456789012345678901234567890121 2
1
2345678901234567890123456789012123456789012345678901234567890121 2
1
2345678901234567890123456789012123456789012345678901234567890121 2
1
2345678901234567890123456789012123456789012345678901234567890121 2
1
2345678901234567890123456789012123456789012345678901234567890121 2
1
2345678901234567890123456789012123456789012345678901234567890121 2
1
2345678901234567890123456789012123456789012345678901234567890121 2
1
2345678901234567890123456789012123456789012345678901234567890121 2
1
2345678901234567890123456789012123456789012345678901234567890121 2
1
2345678901234567890123456789012123456789012345678901234567890121 2
1
2345678901234567890123456789012123456789012345678901234567890121 2
1
2345678901234567890123456789012123456789012345678901234567890121 2
1
2345678901234567890123456789012123456789012345678901234567890121 2
1
2345678901234567890123456789012123456789012345678901234567890121 2
1
2345678901234567890123456789012123456789012345678901234567890121 2
1
2345678901234567890123456789012123456789012345678901234567890121 2
1
2345678901234567890123456789012123456789012345678901234567890121 2
1
2345678901234567890123456789012123456789012345678901234567890121 2
1
2345678901234567890123456789012123456789012345678901234567890121 2
1
2345678901234567890123456789012123456789012345678901234567890121 2
1
2345678901234567890123456789012123456789012345678901234567890121 2
1
2345678901234567890123456789012123456789012345678901234567890121 2
1
2345678901234567890123456789012123456789012345678901234567890121 2
1
2345678901234567890123456789012123456789012345678901234567890121 2
1
2345678901234567890123456789012123456789012345678901234567890121 2
1
2345678901234567890123456789012123456789012345678901234567890121 2
1
2345678901234567890123456789012123456789012345678901234567890121 2
1
2345678901234567890123456789012123456789012345678901234567890121 2
1
2345678901234567890123456789012123456789012345678901234567890121 2
1
2345678901234567890123456789012123456789012345678901234567890121 2
1
2345678901234567890123456789012123456789012345678901234567890121 2
1
2345678901234567890123456789012123456789012345678901234567890121 2
1
2345678901234567890123456789012123456789012345678901234567890121 2
1
2345678901234567890123456789012123456789012345678901234567890121 2
1
2345678901234567890123456789012123456789012345678901234567890121 2
1
2345678901234567890123456789012123456789012345678901234567890121 2
1
2345678901234567890123456789012123456789012345678901234567890121 2
1
2345678901234567890123456789012123456789012345678901234567890121 2
1
2345678901234567890123456789012123456789012345678901234567890121 2
1
2345678901234567890123456789012123456789012345678901234567890121 2
1
2345678901234567890123456789012123456789012345678901234567890121 2
1
2345678901234567890123456789012123456789012345678901234567890121 2
1
2345678901234567890123456789012123456789012345678901234567890121 2
1
2345678901234567890123456789012123456789012345678901234567890121 2
1
2345678901234567890123456789012123456789012345678901234567890121 2
1
2345678901234567890123456789012123456789012345678901234567890121 2
1
2345678901234567890123456789012123456789012345678901234567890121 2
1
12345678901234567890123456789012123456789012345678901234567890121 2
123456789012345678901234567890121234567890123456789012345678901212
LSAT Success
151
www.petersons.com
SUCCESS SESSION 14
123456789012345678901234567890121234567890123456789012345678901212
2345678901234567890123456789012123456789012345678901234567890121 2
1
2345678901234567890123456789012123456789012345678901234567890121 2
1
2345678901234567890123456789012123456789012345678901234567890121 2
1
2345678901234567890123456789012123456789012345678901234567890121 2
1
2345678901234567890123456789012123456789012345678901234567890121 2
1
2345678901234567890123456789012123456789012345678901234567890121 2
1
2345678901234567890123456789012123456789012345678901234567890121 2
1
2345678901234567890123456789012123456789012345678901234567890121 2
1
2345678901234567890123456789012123456789012345678901234567890121 2
1
2345678901234567890123456789012123456789012345678901234567890121 2
1
2345678901234567890123456789012123456789012345678901234567890121 2
1
2345678901234567890123456789012123456789012345678901234567890121 2
1
2345678901234567890123456789012123456789012345678901234567890121 2
1
2345678901234567890123456789012123456789012345678901234567890121 2
1
2345678901234567890123456789012123456789012345678901234567890121 2
1
2345678901234567890123456789012123456789012345678901234567890121 2
1
2345678901234567890123456789012123456789012345678901234567890121 2
1
2345678901234567890123456789012123456789012345678901234567890121 2
1
2345678901234567890123456789012123456789012345678901234567890121 2
1
2345678901234567890123456789012123456789012345678901234567890121 2
1
2345678901234567890123456789012123456789012345678901234567890121 2
1
2345678901234567890123456789012123456789012345678901234567890121 2
1
2345678901234567890123456789012123456789012345678901234567890121 2
1
2345678901234567890123456789012123456789012345678901234567890121 2
1
2345678901234567890123456789012123456789012345678901234567890121 2
1
2345678901234567890123456789012123456789012345678901234567890121 2
1
2345678901234567890123456789012123456789012345678901234567890121 2
1
2345678901234567890123456789012123456789012345678901234567890121 2
1
2345678901234567890123456789012123456789012345678901234567890121 2
1
2345678901234567890123456789012123456789012345678901234567890121 2
1
2345678901234567890123456789012123456789012345678901234567890121 2
1
2345678901234567890123456789012123456789012345678901234567890121 2
1
2345678901234567890123456789012123456789012345678901234567890121 2
1
2345678901234567890123456789012123456789012345678901234567890121 2
1
2345678901234567890123456789012123456789012345678901234567890121 2
1
2345678901234567890123456789012123456789012345678901234567890121 2
1
2345678901234567890123456789012123456789012345678901234567890121 2
1
2345678901234567890123456789012123456789012345678901234567890121 2
1
2345678901234567890123456789012123456789012345678901234567890121 2
1
2345678901234567890123456789012123456789012345678901234567890121 2
1
2345678901234567890123456789012123456789012345678901234567890121 2
1
2345678901234567890123456789012123456789012345678901234567890121 2
1
2345678901234567890123456789012123456789012345678901234567890121 2
1
2345678901234567890123456789012123456789012345678901234567890121 2
1
2345678901234567890123456789012123456789012345678901234567890121 2
1
2345678901234567890123456789012123456789012345678901234567890121 2
1
2345678901234567890123456789012123456789012345678901234567890121 2
1
2345678901234567890123456789012123456789012345678901234567890121 2
1
2345678901234567890123456789012123456789012345678901234567890121 2
1
2345678901234567890123456789012123456789012345678901234567890121 2
1
2345678901234567890123456789012123456789012345678901234567890121 2
1
2345678901234567890123456789012123456789012345678901234567890121 2
1
2345678901234567890123456789012123456789012345678901234567890121 2
1
2345678901234567890123456789012123456789012345678901234567890121 2
1
2345678901234567890123456789012123456789012345678901234567890121 2
1
2345678901234567890123456789012123456789012345678901234567890121 2
1
2345678901234567890123456789012123456789012345678901234567890121 2
1
2345678901234567890123456789012123456789012345678901234567890121 2
1
2345678901234567890123456789012123456789012345678901234567890121 2
1
2345678901234567890123456789012123456789012345678901234567890121 2
1
2345678901234567890123456789012123456789012345678901234567890121 2
1
2345678901234567890123456789012123456789012345678901234567890121 2
1
2345678901234567890123456789012123456789012345678901234567890121 2
1
2345678901234567890123456789012123456789012345678901234567890121 2
1
2345678901234567890123456789012123456789012345678901234567890121 2
1
2345678901234567890123456789012123456789012345678901234567890121 2
1
2345678901234567890123456789012123456789012345678901234567890121 2
1
2345678901234567890123456789012123456789012345678901234567890121 2
1
2345678901234567890123456789012123456789012345678901234567890121 2
1
2345678901234567890123456789012123456789012345678901234567890121 2
1
2345678901234567890123456789012123456789012345678901234567890121 2
1
2345678901234567890123456789012123456789012345678901234567890121 2
1
2345678901234567890123456789012123456789012345678901234567890121 2
1
2345678901234567890123456789012123456789012345678901234567890121 2
1
2345678901234567890123456789012123456789012345678901234567890121 2
1
2345678901234567890123456789012123456789012345678901234567890121 2
1
2345678901234567890123456789012123456789012345678901234567890121 2
1
2345678901234567890123456789012123456789012345678901234567890121 2
1
2345678901234567890123456789012123456789012345678901234567890121 2
1
2345678901234567890123456789012123456789012345678901234567890121 2
1
2345678901234567890123456789012123456789012345678901234567890121 2
1
2345678901234567890123456789012123456789012345678901234567890121 2
1
2345678901234567890123456789012123456789012345678901234567890121 2
1
2345678901234567890123456789012123456789012345678901234567890121 2
1
2345678901234567890123456789012123456789012345678901234567890121 2
1
2345678901234567890123456789012123456789012345678901234567890121 2
1
2345678901234567890123456789012123456789012345678901234567890121 2
1
2345678901234567890123456789012123456789012345678901234567890121 2
1
12345678901234567890123456789012123456789012345678901234567890121 2
123456789012345678901234567890121234567890123456789012345678901212
www.petersons.com
152
LSAT Success
WRITING SAMPLES
123456789012345678901234567890121234567890123456789012345678901212
2345678901234567890123456789012123456789012345678901234567890121 2
1
2345678901234567890123456789012123456789012345678901234567890121 2
1
2345678901234567890123456789012123456789012345678901234567890121 2
1
2345678901234567890123456789012123456789012345678901234567890121 2
1
2345678901234567890123456789012123456789012345678901234567890121 2
1
2345678901234567890123456789012123456789012345678901234567890121 2
1
2345678901234567890123456789012123456789012345678901234567890121 2
1
2345678901234567890123456789012123456789012345678901234567890121 2
1
2345678901234567890123456789012123456789012345678901234567890121 2
1
2345678901234567890123456789012123456789012345678901234567890121 2
1
2345678901234567890123456789012123456789012345678901234567890121 2
1
2345678901234567890123456789012123456789012345678901234567890121 2
1
2345678901234567890123456789012123456789012345678901234567890121 2
1
2345678901234567890123456789012123456789012345678901234567890121 2
1
2345678901234567890123456789012123456789012345678901234567890121 2
1
2345678901234567890123456789012123456789012345678901234567890121 2
1
2345678901234567890123456789012123456789012345678901234567890121 2
1
2345678901234567890123456789012123456789012345678901234567890121 2
1
2345678901234567890123456789012123456789012345678901234567890121 2
1
2345678901234567890123456789012123456789012345678901234567890121 2
1
2345678901234567890123456789012123456789012345678901234567890121 2
1
2345678901234567890123456789012123456789012345678901234567890121 2
1
2345678901234567890123456789012123456789012345678901234567890121 2
1
2345678901234567890123456789012123456789012345678901234567890121 2
1
2345678901234567890123456789012123456789012345678901234567890121 2
1
2345678901234567890123456789012123456789012345678901234567890121 2
1
2345678901234567890123456789012123456789012345678901234567890121 2
1
2345678901234567890123456789012123456789012345678901234567890121 2
1
2345678901234567890123456789012123456789012345678901234567890121 2
1
2345678901234567890123456789012123456789012345678901234567890121 2
1
2345678901234567890123456789012123456789012345678901234567890121 2
1
2345678901234567890123456789012123456789012345678901234567890121 2
1
2345678901234567890123456789012123456789012345678901234567890121 2
1
2345678901234567890123456789012123456789012345678901234567890121 2
1
2345678901234567890123456789012123456789012345678901234567890121 2
1
2345678901234567890123456789012123456789012345678901234567890121 2
1
2345678901234567890123456789012123456789012345678901234567890121 2
1
2345678901234567890123456789012123456789012345678901234567890121 2
1
2345678901234567890123456789012123456789012345678901234567890121 2
1
2345678901234567890123456789012123456789012345678901234567890121 2
1
2345678901234567890123456789012123456789012345678901234567890121 2
1
2345678901234567890123456789012123456789012345678901234567890121 2
1
2345678901234567890123456789012123456789012345678901234567890121 2
1
2345678901234567890123456789012123456789012345678901234567890121 2
1
2345678901234567890123456789012123456789012345678901234567890121 2
1
2345678901234567890123456789012123456789012345678901234567890121 2
1
2345678901234567890123456789012123456789012345678901234567890121 2
1
2345678901234567890123456789012123456789012345678901234567890121 2
1
2345678901234567890123456789012123456789012345678901234567890121 2
1
2345678901234567890123456789012123456789012345678901234567890121 2
1
2345678901234567890123456789012123456789012345678901234567890121 2
1
2345678901234567890123456789012123456789012345678901234567890121 2
1
12345678901234567890123456789012123456789012345678901234567890121 2
123456789012345678901234567890121234567890123456789012345678901212
LSAT Success
153
www.petersons.com
SUCCESS SESSION 14
123456789012345678901234567890121234567890123456789012345678901212
2345678901234567890123456789012123456789012345678901234567890121 2
1
2345678901234567890123456789012123456789012345678901234567890121 2
1
2345678901234567890123456789012123456789012345678901234567890121 2
1
2345678901234567890123456789012123456789012345678901234567890121 2
1
2345678901234567890123456789012123456789012345678901234567890121 2
1
2345678901234567890123456789012123456789012345678901234567890121 2
1
2345678901234567890123456789012123456789012345678901234567890121 2
1
2345678901234567890123456789012123456789012345678901234567890121 2
1
2345678901234567890123456789012123456789012345678901234567890121 2
1
2345678901234567890123456789012123456789012345678901234567890121 2
1
2345678901234567890123456789012123456789012345678901234567890121 2
1
2345678901234567890123456789012123456789012345678901234567890121 2
1
2345678901234567890123456789012123456789012345678901234567890121 2
1
2345678901234567890123456789012123456789012345678901234567890121 2
1
2345678901234567890123456789012123456789012345678901234567890121 2
1
2345678901234567890123456789012123456789012345678901234567890121 2
1
2345678901234567890123456789012123456789012345678901234567890121 2
1
2345678901234567890123456789012123456789012345678901234567890121 2
1
2345678901234567890123456789012123456789012345678901234567890121 2
1
2345678901234567890123456789012123456789012345678901234567890121 2
1
2345678901234567890123456789012123456789012345678901234567890121 2
1
2345678901234567890123456789012123456789012345678901234567890121 2
1
2345678901234567890123456789012123456789012345678901234567890121 2
1
2345678901234567890123456789012123456789012345678901234567890121 2
1
2345678901234567890123456789012123456789012345678901234567890121 2
1
2345678901234567890123456789012123456789012345678901234567890121 2
1
2345678901234567890123456789012123456789012345678901234567890121 2
1
2345678901234567890123456789012123456789012345678901234567890121 2
1
2345678901234567890123456789012123456789012345678901234567890121 2
1
2345678901234567890123456789012123456789012345678901234567890121 2
1
2345678901234567890123456789012123456789012345678901234567890121 2
1
2345678901234567890123456789012123456789012345678901234567890121 2
1
2345678901234567890123456789012123456789012345678901234567890121 2
1
2345678901234567890123456789012123456789012345678901234567890121 2
1
2345678901234567890123456789012123456789012345678901234567890121 2
1
2345678901234567890123456789012123456789012345678901234567890121 2
1
2345678901234567890123456789012123456789012345678901234567890121 2
1
2345678901234567890123456789012123456789012345678901234567890121 2
1
2345678901234567890123456789012123456789012345678901234567890121 2
1
2345678901234567890123456789012123456789012345678901234567890121 2
1
2345678901234567890123456789012123456789012345678901234567890121 2
1
2345678901234567890123456789012123456789012345678901234567890121 2
1
2345678901234567890123456789012123456789012345678901234567890121 2
1
2345678901234567890123456789012123456789012345678901234567890121 2
1
2345678901234567890123456789012123456789012345678901234567890121 2
1
2345678901234567890123456789012123456789012345678901234567890121 2
1
2345678901234567890123456789012123456789012345678901234567890121 2
1
2345678901234567890123456789012123456789012345678901234567890121 2
1
2345678901234567890123456789012123456789012345678901234567890121 2
1
2345678901234567890123456789012123456789012345678901234567890121 2
1
2345678901234567890123456789012123456789012345678901234567890121 2
1
2345678901234567890123456789012123456789012345678901234567890121 2
1
2345678901234567890123456789012123456789012345678901234567890121 2
1
2345678901234567890123456789012123456789012345678901234567890121 2
1
2345678901234567890123456789012123456789012345678901234567890121 2
1
2345678901234567890123456789012123456789012345678901234567890121 2
1
2345678901234567890123456789012123456789012345678901234567890121 2
1
2345678901234567890123456789012123456789012345678901234567890121 2
1
2345678901234567890123456789012123456789012345678901234567890121 2
1
2345678901234567890123456789012123456789012345678901234567890121 2
1
2345678901234567890123456789012123456789012345678901234567890121 2
1
2345678901234567890123456789012123456789012345678901234567890121 2
1
2345678901234567890123456789012123456789012345678901234567890121 2
1
2345678901234567890123456789012123456789012345678901234567890121 2
1
2345678901234567890123456789012123456789012345678901234567890121 2
1
2345678901234567890123456789012123456789012345678901234567890121 2
1
2345678901234567890123456789012123456789012345678901234567890121 2
1
2345678901234567890123456789012123456789012345678901234567890121 2
1
2345678901234567890123456789012123456789012345678901234567890121 2
1
2345678901234567890123456789012123456789012345678901234567890121 2
1
2345678901234567890123456789012123456789012345678901234567890121 2
1
2345678901234567890123456789012123456789012345678901234567890121 2
1
2345678901234567890123456789012123456789012345678901234567890121 2
1
2345678901234567890123456789012123456789012345678901234567890121 2
1
2345678901234567890123456789012123456789012345678901234567890121 2
1
2345678901234567890123456789012123456789012345678901234567890121 2
1
2345678901234567890123456789012123456789012345678901234567890121 2
1
2345678901234567890123456789012123456789012345678901234567890121 2
1
2345678901234567890123456789012123456789012345678901234567890121 2
1
2345678901234567890123456789012123456789012345678901234567890121 2
1
2345678901234567890123456789012123456789012345678901234567890121 2
1
2345678901234567890123456789012123456789012345678901234567890121 2
1
2345678901234567890123456789012123456789012345678901234567890121 2
1
2345678901234567890123456789012123456789012345678901234567890121 2
1
2345678901234567890123456789012123456789012345678901234567890121 2
1
2345678901234567890123456789012123456789012345678901234567890121 2
1
2345678901234567890123456789012123456789012345678901234567890121 2
1
2345678901234567890123456789012123456789012345678901234567890121 2
1
12345678901234567890123456789012123456789012345678901234567890121 2
123456789012345678901234567890121234567890123456789012345678901212
www.petersons.com
154
LSAT Success
Success Session 15
FINISH WITH A FLOURISH
In this final session, you will put the finishing
touches on your LSAT training techniques. The
first part of the session briefly reviews the critical
test performance factors you have been concentrating on. If you are uncertain about any of the
test-taking techniques as you work through the
review, stop and go back to the session that deals
with the subject. Work through the techniques
until your uncertainty is put to rest.
The second part of the session alerts you to
a variety of minor factors that also influence your
test performance and discusses quick-and-easy
techniques for controlling them.
WHAT
The Techniques
PERFORMANCE FACTORS?
Anxiety
Preparation anxiety and test anxiety influence
your performance. Control is the key to managing them. You gain control by choosing to take
the test, familiarizing yourself with it, thinking
positively, and worrying constructively. You
choose to accept the LSAT challenge and all of
the baggage that travels with it. You make
yourself familiar with the test and its use in every
practical detail. You develop techniques that
allow you to have a positive response to every
aspect of the LSAT. You recognize worry as the
sign of a loss of control, which triggers a
response that will work to regain that control.
Discipline
Discipline is required for you to develop your
section-test plan. Maintaining the discipline of
working through the LSAT by question sets is the
keystone of your performance. The discipline of
completing the pretest question sets provides
you with the information upon which you base
your plan for optimum performance (highest
score). Having the discipline to consistently
execute the techniques within the context of
your plan maximizes your performance.
Conditioning
Conditioning is even more important to your test
performance than anxiety. Nearly all of your
former academic and test-taking conditioning is
potentially detrimental to your LSAT performance. You have neither the time nor reason to
alter it. Your strategy is to avoid your conditioning and, as a consequence, minimize its influence
on your performance. Learning specific techniques for handling every aspect of the test
renders your conditioning irrelevant to the LSAT.
WHAT
TEST IS IMMINENT?
The Date
You choose the date on which you will take the
LSAT. The most important factor to consider
when making the choice is readiness. Use the
155
SUCCESS SESSION 15
WHAT
TEST IS TOMORROW?
The Tools
You will need some obvious and not-so-obvious
tools to take the LSAT. Get them together early
on the day before the test. The No. 2 pencil is
tool number one, as answer sheets must be
completed in pencil. Prepare and take four
pencils to the testthey should have full erasers
and not be needle sharp. (Needle-sharp pencils
lacerate answer sheets, and their points splinter.
Slightly blunted points are best.) The Writing
Sample must be completed in pen, and pens are
provided at the test. Though it is not explicitly
sanctioned, many test-takers prefer to use their
own pens. They should have black ink and be
erasable, which eliminates the need for crossing
out. The resulting neat look is a plus in ensuring
that your Writing Sample does not disqualify you.
If you wish to use a highlighter for marking
Passages or Arguments problems, bring at least
two in the colors of your choice.
A reliable watch or equivalent time-minder
is essential to your performance. Before you
begin to work through a section-test, calculate
and write at the starting point of each question
set within it the exact time you have planned to
be at that point. This makes your time checks
easy to perform as you track your pace through
each question set.
The test-maker discourages the ingestion of
food at the test center, but the test is long, and
administrative procedures at some centers make
the testing time even longer. How does a conscientious test-taker reconcile a desire not to transgress by eating with the equally strong desire not
to have a growling stomach disturb the test center? Those who have resolved this dilemma successfully strongly advise against the crunchy, the
smelly, and the gooey. Pretzels, potato chips, or
celery sticks crunch noisily when eaten. Worse
still are beef jerky, pepperoni, and cheese. Their
capacity for olfactory offense is nearly limitless.
Avoid fudge, toffee, nougat, and the like, since
they are sticky and endanger neat paperwork. Otherwise, it is the test-takers choicecookies and
candy work well. During cool weather, jacket
pockets are good for holding food. The marsupial
sweatshirt with the pouch in the front carries a
lot, is comfortable, and gives convenient access to
your food. Make sure you snack between, not during, question sets.
The Site
Register to take the test at a site that gives you
an advantage. For many, the home-court
advantage is a major performance plus. Others
trade familiarity for small size. A huge test site
with central check-in, room assignments, and
other complexities can trash a large reserve of
control in short order, whereas a small center
sometimes provides the advantages of informality
and low tension. Find out about the location and
size of test centers from Law School Admission
Services if you think that either might give you
an advantage.
Wherever you choose to take the LSAT,
familiarize yourself with the site in advance.
Know exactly how you will travel there and
what you will do with your car when you arrive.
Many a person has lost test-taker control to
campus parking, rearranged for a homecoming
football game. If you cannot visit the site before
the test date, get there early and check it out.
Then leave the test area and return just before
test time.
Locate the rest room at the test site and the
most direct route to it from the test room. This is
important since, if you have to use the rest room
during the test, it is on your time. There is
usually a 15-minute break between the second
and third section-tests.
Light is also a matter of concern. Trying to
concentrate on the LSAT when you are sitting in
a dark corner of an auditorium or staring into the
sun is not advantageous. If the light at your
assigned seat is inadequate, seek out the proctor
and insist, politely but firmly, that you be moved.
Avoid distractions. If you are easily distracted, you should not sit at the back of a room
of 200 people, each of whom will provide you
with at least one distraction in the course of the
test. For you, its down in front.
www.petersons.com
156
LSAT Success
The Run-Up
Two basic strategies are advocated for the time
just before the test. Advocates of the Jell-O
strategy believe that it is best to sit back and let
your plan and techniques gel during this period.
Ben-Gay advocates claim that a little warm-up
working one last set of questions is the best way
to spend it. Studies have shown that neither
strategy has a greater impact than the other on
performance.
What is significant is that you avoid either
zoom or doom and gloom. Extremes of
emotion and activity can have a negative impact
on your performance. Maintaining emotional
equilibrium and your typical level of activity will
ensure that you are in the best position to
concentrate during the test. Follow a regimen
that encourages a sense of relaxation. If a good
party the night before is a shortcut to relaxation
for you, try to avoid relaxing too far. There is no
evidence that a roaring hangover leads to poor
test performance, but the LSAT is hard enough
without helping it along.
Adjust your personal schedule to the hour
of the test. If you are not a morning person and
are scheduled to take the LSAT in the morning,
set your alarm to give you sufficient time to
come out of your morning fog before the test
starts.
WHAT
TEST IS NOW?
The Routines
On the day of the test, proceed as normally as
possible, especially with respect to your schedule, level of activity, and intake of food and
liquid. Staying as close as possible to your regular
schedule supports you in carrying out your
planned response to the predictability of the
LSAT. Changing all of your routines in honor of
test day is counterproductive, since it disrupts
your plan. Eating a lumberjacks breakfast on the
morning of the test in order to avoid the risk of
midmorning burnout is more likely to give you
LSAT Success
157
www.petersons.com
SUCCESS SESSION 15
enhance your identification of wrong answers
and your test performance.
The first alert relates to the test-makers use
of absolutes and qualifiersalways, never, some,
few, all, none, every, many, and similar terms
usually flag a pivotal or decisive condition. When
you encounter an absolute or a qualifier in a
statement or answer option, focus your full
attention on its implications within the question
context. It is not coincidentalthe use of an
absolute or qualifier is intentional and significant.
The test-makers use of negatives and
exceptions is frequently an obfuscation strategy.
Be alert to NOT, EXCEPT, and their progeny and
relatives (they are usually capitalized, as shown
here). The rate of best-answer selection falls
substantially for many test-takers when negatives
and exceptions are involved. Because the large
majority of LSAT statements, questions, and
answer options involve positives, you anticipate a
positive and often overlook a negative. So, when
you first see a negative or an exception in a
statement, question, or answer option, note it
boldly to keep yourself alert to the point as you
work through the question.
When a double negative appearsa NOT in
the statement and a CANNOT in the question, or a
CANNOT in the question and a NOT in an answer
optiona double alert is called for. A similar
double alert applies to the combination of a negative and an exception. For example, such questions as All of the following are not true EXCEPT.
. . . require redoubled concentration for you.
The mismatched answer-option structure
also deserves an alert. When you encounter an
answer-option set in which four of the options
present a similar word pattern and the fifth is totally different, your eye is drawn to the different
one as a possible answer. It usually is not.
Be alert to questions and answer options that
appear later in a series based on a single statement. You may often be attracted to an answer
because your thinking has been influenced by the
preceding questions and answer options. If you
find yourself falling prey to this phenomenon, try
considering questions and answer options out of
the sequence in which they appear. Work through
the last first or the middle last.
Be especially alert to the multiple-option
format. Recall that it appears to be more difficult
because more than one of the Roman numeral
options can satisfy the required conditions. For
most test-takers, performance suffers on questions employing this format.
Assorted Alerts
As your work in previous sessions has shown
you, the techniques designed to neutralize the
test-makers obfuscation patterns and strategies
simultaneously facilitate wrong-answer identification and superior test performance. There are
also a number of patterns and strategies that the
test-maker uses episodically with all LSAT
question types. Being alert to these further
patterns and strategies while applying the other
test-taking techniques you have learned will
www.petersons.com
158
LSAT Success
HOW
DECISIONS?
Debriefing
Immediately upon completion of the test and
before you leave the test center, take a few
minutes to note particulars of the test and the
administration that you are able to remember.
For future reference, record the order of the
section-tests, the number of question sets you
completed, special difficulties encountered,
surprises, uncertainties, and the like. At some
point during the next 24 hours, review these
notes. It is desirable to conduct this debriefing
with someone elsepreferably someone familiar
with the LSAT.
The objectives of this debriefing are to
ascertain any significant discrepancies between
your test plan and your actual performance and
to identify any factors that had a negative impact
on your performance. If you deviated significantly from your test plan or encountered a
problem that you are convinced means you did
not perform well on the test, the possibility of
canceling your test score arises.
Postmortems
Reviewing test questions, answer selections, and
the like with fellow test-takers seldom provides
any reliable information about your performance.
In fact, the typical LSAT postmortem tends to
confound posttest decision making. The prospect
of getting any benefit from a postmortem is so
slight that you should make every effort to avoid
such after-test comparisons. Stick with the
debriefing as the most effective way of making
informed posttest decisions.
Retakes
There is no restriction on the number of times
you may take the LSAT. Of course, totally
different questions are used at each administration of the test. And the average of an applicants
test scores is reported to law schools in addition
to the scores themselves.
Studies done over a period of many years
show that mostbut certainly not allpeople
who retake the LSAT do score higher on the
retake. The same studies show that the increase
is small for the majority, and the score increase
appears to be even smaller when averaged. For
example, a score improvement of four points on
a retake results in an averaged score increase of
only two points.
Only when your pretest data indicate that a
large score improvement could be realized is a
retake recommended. From the perspective of
most admission committees, the reflection of a
very small or nonexistent score increase seems to
cast a shadow rather than a glow on an applicant.
Cancellation
Once the test administration has begun, your
options with respect to the test score are to do
nothing or to cancel your score. If you do
nothing, your answer sheet is scored and the
score reported. If you cancel your score, it is not
reported, but the fact that the score was
canceled is reported. Thus, you cancel your
score only when the debriefing produces solid
information that an informed person other than
yourself interprets as a significant deviation from
your test plan or a serious problem. A mere
sense of unease about your performance or a
feeling that you did not perform as well as you
might have does not constitute a clear and
convincing reason to cancel your score.
You have five working days to cancel your
test score. There is no reason to rush to cancel
with less than full information and an informed
perspective to guide your decision. If you do
decide to cancel, you must do so in writing to
LSAS, Box 2000-T, Newtown, PA 18940. A
mailgram, telegram, or letter requesting that your
score be canceled is sufficient. (At the time of
this writing, a request by fax does not meet the
stated requirements of LSAS.) You do not need to
explain your decision, so dont.
LSAT Success
WHAT
The Wait
The wait for test results is relatively long. It
generally takes between four and six weeks for
results to be reported. The test-maker waits for
nearly all answer sheets to be received, processed, and checked before issuing scores. There
is no way to hurry the process along.
159
www.petersons.com
SUCCESS SESSION 15
The Check
The Response
www.petersons.com
ARE
ADVICE?
160
LSAT Success
161
www.petersons.com
162
LSAT Success
Practice Test 1
SECTION 1
TIME35
Questions 16
A student is preparing a report on statehood. The
source material is incomplete, but the following
is known.
Wyoming became a state before Ohio.
Kansas became a state before Wyoming.
Ohio became a state after Maine.
(A)
(B)
(C)
(D)
(E)
6. If Alaska became a state after Iowa and Wyoming, which of the following must be true?
(A)
(B)
(C)
(D)
(E)
24 QUESTIONS
(A)
(B)
(C)
(D)
(E)
MINUTES
163
PRACTICE TEST 1
12. If the number of males related to T equals
the number of females related to T, which
of the following can be true?
Questions 712
T lives in a smaller house than her brother.
T lives in a larger house than her parents.
Ts children live with T.
T has no other relatives.
(A)
(B)
(C)
(D)
(E)
1,
0,
2,
1,
2,
father to son
mother to daughter
daughter to mother
grandfather to grandson
son to father
children.
children.
child.
children.
children.
S is Us son
S is Us mother
U is younger than S
S is younger than U
U and S are both female
A, D, G
D, E, F
D, E, G
E, F, G
B, D, G
grandson
uncle
nephew
son
granddaughter
1 only
2 only
3 only
2 and 3 only
1, 2, and 3
4
3
1
6
2
exactly
exactly
exactly
exactly
exactly
has
has
has
has
has
Questions 1318
0
1
1
2
0
T
T
T
T
T
3
4
5
6
7
less than 2
2
2 or 3
3
more than 3
www.petersons.com
164
LSAT Success
SECTION 1
20. Which of the following must be true?
1
2
3
1
2
only
only
only
and 3 only
and 3 only
0
1
2
3
4
A, C, E, G
A, D, E, G
A, E, F, G
B, D, E, F
B, D, E, G
Questions 1924
Angela, Bruce, Cora, Dora, and Elmer live at
different points along a straight east-west
highway.
LSAT Success
10 miles
8 miles
5 miles
4 miles
2 miles
(A)
(B)
(C)
(D)
(E)
165
14
15
16
17
18
www.petersons.com
PRACTICE TEST 1
SECTION 2
TIME35
24 QUESTIONS
www.petersons.com
MINUTES
166
LSAT Success
SECTION 2
5. The United States gets 5 percent of its oil
from Mexico. If Mexico raises the price of
its oil by 20 percent, that will result in an
increase of 1 percent (5 percent times 20
percent) in the price of oil products in the
United States.
7. The policy of equal pay for women continues to erode the importance of the mothers
role in society.
The above argument can be criticized for
which of the following reasons?
(A) The importance of a role is not related
to the pay for that role.
(B) Equal pay for women is unrelated to
motherhood.
(C) All women are not mothers.
(D) Society continues to devalue motherhood.
(E) When someone gains in a society,
someone else loses.
LSAT Success
167
www.petersons.com
PRACTICE TEST 1
11. Which of the following can be inferred
from the paragraph above?
Questions 910
Lecturer: On average, the majority of Americans
enjoy the highest standard of living of any people
in the world.
10. A logical criticism of the lecturers statement would focus on the existence of
(A) a country in which the majority of
people enjoy a higher standard of
living than that of the American
people
(B) a country with a higher level of
employment than America
(C) poor Americans who receive federal
aid
(D) a higher level of inflation in America
than in other countries
(E) many poor American families that are
so isolated that they are not included
in statistical surveys
Questions 1112
The position that the prohibition of morally
offensive works is wrong in principle is hardly
tenable. There certainly are circumstances in
which censorship could be desirable. If it were
shown that all or most people of a certain type
who saw a film thereafter committed a burglary
or murder that they would not otherwise have
committed, no one would deny that public
exhibition of the film should be prohibited. To
admit this is to admit that censorship is not
wrong in principle. But to approve the principle
of censorship on these grounds does not, of
course, commit one to approve censorship in
every form.
www.petersons.com
168
LSAT Success
SECTION 2
14. Many people confuse reasons and causes.
Any justification for performing an action is
a reason. Anything that makes performing
an action necessary is a causefor example,
a strong urge, hunger, an intense desire,
social pressure, or some brain disorder.
Those people who believe that the same
thing may be both a reason for performing
an action and its cause are clearly mistaken.
hunger
some brain disorder
social pressure
a strong urge
an intense desire
LSAT Success
169
www.petersons.com
PRACTICE TEST 1
18. During the cultural revolution in China
under Chairman Mao, thousands of enemies of the republic were killed. When
Maos critics accused him of confusing his
personal enemies with enemies of the
republic, he responded, I deny the
accusation, and the proof is that you are
still alive.
www.petersons.com
170
LSAT Success
SECTION 2
22. When pregnant laboratory rats are given
caffeine equivalent to the amount a human
would consume by drinking six cups of
coffee per day, an increase in the incidence
of birth defects results. When asked if the
government would require warning labels
on products containing caffeine, a spokesperson stated that it would not, because if
the finding of these studies were to be
refuted in the future, the government would
lose credibility.
LSAT Success
171
www.petersons.com
PRACTICE TEST 1
SECTION 3
TIME35
(A)
(B)
(C)
(D)
(E)
Questions 16
(A)
(B)
(C)
(D)
(E)
(A)
(B)
(C)
(D)
(E)
located
located
located
located
located
due north of G.
west of E.
east of B.
due west of C.
due north of E.
southwest of A
northeast of C
southeast of E
southeast of G
northwest of G
northwest corner of D
southeast corner of G
northwest corner of A
west side of A
east side of G
www.petersons.com
is
is
is
is
is
the
the
the
the
the
F
F
F
F
F
(A)
(B)
(C)
(D)
(E)
24 QUESTIONS
MINUTES
172
LSAT Success
SECTION 3
9. If J, the secretary, must sit across from one
of the trustees, how might the officers be
arranged clockwise in order to satisfy all
conditions?
Questions 712
Six college officersH, I, J, K, L, and Mare
seated at equal distances around a circular table
according to a list of personal preferences
submitted by each officer.
(A)
(B)
(C)
(D)
(E)
(A)
(B)
(C)
(D)
(E)
(A)
(B)
(C)
(D)
(E)
I and K
H and J
H and K
I and J
H and L
(A)
(B)
(C)
(D)
(E)
LSAT Success
J, K, L
I, J, L
K, J, I
K, L, I
L, J, K
L, M, I, K, J, H
K, M, J, L, H, I
J, K, L, H, M, I
K, I, H, J, M, L
I, J, K, M, H, L
173
M is the secretary
H is not the treasurer
J is not the vice president
I is a trustee
either J or K is a trustee
www.petersons.com
PRACTICE TEST 1
16. Which of the following is NOT a possible
order in which the vehicles are washed?
Questions 1318
Holly Hauling has six vehicles. The Kenworth,
Mack, and White are trucks; the Chevrolet,
Dodge, and Ford are vans.
Dodge,
Chevrolet,
Ford, Dodge,
Dodge, Ford,
(A)
(B)
(C)
(D)
(E)
Ford,
White, Mack,
White,
Kenworth,
Kenworth,
Mack, White,
The
The
The
The
The
www.petersons.com
174
LSAT Success
SECTION 3
21. If Group 1 is concrete, Group 3 must be
which of the following?
Questions 1924
There are six distinct building groups in a large
office complex. From smallest to largest,
respectively, the groups are constructed of
aluminum, brick, concrete, glass, stone, and
wood. The building groups are designated
Groups 1 through 6.
(A)
(B)
(C)
(D)
(E)
(A)
(B)
(C)
(D)
(E)
aluminum
brick
concrete
glass
stone
5,
6,
6,
6,
6,
LSAT Success
3,
3,
3,
3,
5,
6,
1,
1,
5,
3,
1,
5,
2,
2,
2,
(A)
(B)
(C)
(D)
(E)
2,
2,
5,
1,
1,
5,
5,
6,
6,
6,
6,
6,
5,
5,
5,
3,
3,
2,
3,
4,
1,
2,
4,
1,
3,
2,
4,
3,
2,
2,
4
1
1
4
1
aluminum
brick
glass
stone
wood
concrete
stone
wood
brick
aluminum
4
4
4
4
4
175
concrete
glass
brick
aluminum
wood
www.petersons.com
PRACTICE TEST 1
SECTION 4
TIME35
MINUTES
45
50
Line
10
15
20
25
30
35
40
www.petersons.com
55
60
65
70
28 QUESTIONS
176
LSAT Success
SECTION 4
2. According to the passage, all of the following are methods used to explain social
policy EXCEPT
(A)
(B)
(C)
(D)
(E)
the
the
the
the
the
Scottish method
theory of welfare economics
perfect-competition paradigm
scientific method
principles of natural liberty
Line
15
20
25
LSAT Success
177
www.petersons.com
PRACTICE TEST 1
30
35
40
45
50
55
60
65
70
75
www.petersons.com
178
LSAT Success
SECTION 4
15
20
25
30
legal reform
false accusations
results of human inaction
means used to produce results
aspirations producing human action
35
40
45
10
somewhat critical
generally supportive
mostly accepting
totally convinced
nearly convinced
50
55
LSAT Success
60
65
179
www.petersons.com
PRACTICE TEST 1
15. The passage can best be summarized as
(A)
(B)
(C)
(D)
as a method of argumentation
as a source of answers to problems
as a form of reasoning used by lawyers
as a method to identify problems with
logic
(E) as a technique to ensure equity
Line
is
is
is
is
is
10
15
20
www.petersons.com
180
LSAT Success
SECTION 4
25
30
35
40
45
50
55
60
65
70
LSAT Success
A, 2, #, +, s, ?, c, p, %, $
A, C, A, E, A, G, A, I, A, K
1, 2, 3, 4, 5, 6, 5, 4, 3, 2, 1
@, %, &, @, %, &, @, %, &
8, 1, 9, 1, 7, 1, 6, 1, 5, 1, 4, 1
181
www.petersons.com
PRACTICE TEST 1
27. According to the passage, which of the
following is NOT true?
www.petersons.com
182
LSAT Success
SECTION 5
SECTION 5
TIME35 MINUTES
(A) a way of life consistent with benevolent ideals is possible in the modern
world
(B) people can be persuaded to abandon
technology, urbanization, and mass
production
(C) benevolent conduct can result from
humans living in accordance with
their own natural dispositions
(D) benevolent dispositions give rise to
evil institutions
(E) corrupt institutions can be eliminated
or reformed
LSAT Success
24 QUESTIONS
183
www.petersons.com
PRACTICE TEST 1
5. Only excellent musicians can be professors at
Juilliard. No insensitive people are great lovers of poetry. No one who is not sensitive
can be a lover. There are no excellent musicians who are not great lovers of poetry.
Therefore, all Juilliard professors are lovers.
www.petersons.com
184
LSAT Success
SECTION 5
10. By 1997, the number of 18-year-olds will be
dramatically lower than it was in 1961,
when population growth in the United
States reached its highest point. This decline
in the number of potential college students
will result in large enrollment decreases at
colleges in the United States.
Questions 1213
Critics who claim that the sale of U.S. military
equipment to other countries is destabilizing and
leads to war take a narrow view of history. War
occurs when one country gains a military
advantage over another. By selling arms, the
United States can ensure that the military balance
among countries is maintained and war avoided.
LSAT Success
185
www.petersons.com
PRACTICE TEST 1
16. Which of the following best expresses the
underlying point of the above argument?
(A)
(B)
(C)
(D)
Questions 1516
The publics right to know is an inadequate
justification for exposing peoples private lives to
public scrutiny. Only when the public welfare is
involved does the public have a right to know
information about a persons private life.
Questions 1819
A survey of students concludes that some
students prefer physics to history; all students
prefer history to geometry; no students prefer
history to economics; and all students prefer
biology to history.
www.petersons.com
18. Based on the survey results, which of the following must represent students preferences?
(A) Some students
physics.
(B) Some students
geometry.
(C) Some students
economics.
(D) Some students
economics.
(E) Some students
economics.
186
prefer geometry to
prefer physics to
prefer biology to
prefer geometry to
prefer physics to
LSAT Success
SECTION 5
19. Based on the survey results, which of the
following CANNOT represent students
preferences?
(A) Some students
physics.
(B) Some students
geometry.
(C) Some students
economics.
(D) Some students
biology.
(E) Some students
economics.
prefer geometry to
prefer physics to
prefer biology to
prefer economics to
prefer physics to
LSAT Success
187
www.petersons.com
PRACTICE TEST 1
24. Spring Lake does not appear to be good for
sailing. I have gone to the lake many times
this year, and each time the water was too
rough for sailing.
Which of the following most closely
parallels the above argument?
(A) It appears that we will move to Spring
Lake this year. The city is simply too
rough for safe living.
(B) Economy-grade gasoline apparently
does not prevent my car from running
rough. It appears that a good grade of
fuel is required.
(C) It appears that the cost of housing at
Spring Lake is prohibitive. I looked at
a number of houses last month, and
they cost much more than I could
afford.
(D) I am withdrawing from Spring Lake
College. Two months at school was
sufficient to prove that college was
too rough for me.
(E) It appears that I will never play the
clarinet. I began lessons many times,
but each time, I quit.
www.petersons.com
188
LSAT Success
123456789012345678901234567890121234567890123456789012345678901212
2345678901234567890123456789012123456789012345678901234567890121 2
1
2345678901234567890123456789012123456789012345678901234567890121 2
1
2345678901234567890123456789012123456789012345678901234567890121 2
1
2345678901234567890123456789012123456789012345678901234567890121 2
1
2345678901234567890123456789012123456789012345678901234567890121 2
1
2345678901234567890123456789012123456789012345678901234567890121 2
1
2345678901234567890123456789012123456789012345678901234567890121 2
1
2345678901234567890123456789012123456789012345678901234567890121 2
1
2345678901234567890123456789012123456789012345678901234567890121 2
1
2345678901234567890123456789012123456789012345678901234567890121 2
1
2345678901234567890123456789012123456789012345678901234567890121 2
1
2345678901234567890123456789012123456789012345678901234567890121 2
1
2345678901234567890123456789012123456789012345678901234567890121 2
1
2345678901234567890123456789012123456789012345678901234567890121 2
1
2345678901234567890123456789012123456789012345678901234567890121 2
1
2345678901234567890123456789012123456789012345678901234567890121 2
1
2345678901234567890123456789012123456789012345678901234567890121 2
1
2345678901234567890123456789012123456789012345678901234567890121 2
1
2345678901234567890123456789012123456789012345678901234567890121 2
1
2345678901234567890123456789012123456789012345678901234567890121 2
1
2345678901234567890123456789012123456789012345678901234567890121 2
1
2345678901234567890123456789012123456789012345678901234567890121 2
1
2345678901234567890123456789012123456789012345678901234567890121 2
1
2345678901234567890123456789012123456789012345678901234567890121 2
1
2345678901234567890123456789012123456789012345678901234567890121 2
1
12345678901234567890123456789012123456789012345678901234567890121 2
123456789012345678901234567890121234567890123456789012345678901212
LSAT Success
189
www.petersons.com
PRACTICE TEST 1
123456789012345678901234567890121234567890123456789012345678901212
2345678901234567890123456789012123456789012345678901234567890121 2
1
2345678901234567890123456789012123456789012345678901234567890121 2
1
2345678901234567890123456789012123456789012345678901234567890121 2
1
2345678901234567890123456789012123456789012345678901234567890121 2
1
2345678901234567890123456789012123456789012345678901234567890121 2
1
2345678901234567890123456789012123456789012345678901234567890121 2
1
2345678901234567890123456789012123456789012345678901234567890121 2
1
2345678901234567890123456789012123456789012345678901234567890121 2
1
2345678901234567890123456789012123456789012345678901234567890121 2
1
2345678901234567890123456789012123456789012345678901234567890121 2
1
2345678901234567890123456789012123456789012345678901234567890121 2
1
2345678901234567890123456789012123456789012345678901234567890121 2
1
2345678901234567890123456789012123456789012345678901234567890121 2
1
2345678901234567890123456789012123456789012345678901234567890121 2
1
2345678901234567890123456789012123456789012345678901234567890121 2
1
2345678901234567890123456789012123456789012345678901234567890121 2
1
2345678901234567890123456789012123456789012345678901234567890121 2
1
2345678901234567890123456789012123456789012345678901234567890121 2
1
2345678901234567890123456789012123456789012345678901234567890121 2
1
2345678901234567890123456789012123456789012345678901234567890121 2
1
2345678901234567890123456789012123456789012345678901234567890121 2
1
2345678901234567890123456789012123456789012345678901234567890121 2
1
2345678901234567890123456789012123456789012345678901234567890121 2
1
2345678901234567890123456789012123456789012345678901234567890121 2
1
2345678901234567890123456789012123456789012345678901234567890121 2
1
2345678901234567890123456789012123456789012345678901234567890121 2
1
2345678901234567890123456789012123456789012345678901234567890121 2
1
2345678901234567890123456789012123456789012345678901234567890121 2
1
2345678901234567890123456789012123456789012345678901234567890121 2
1
2345678901234567890123456789012123456789012345678901234567890121 2
1
2345678901234567890123456789012123456789012345678901234567890121 2
1
2345678901234567890123456789012123456789012345678901234567890121 2
1
2345678901234567890123456789012123456789012345678901234567890121 2
1
2345678901234567890123456789012123456789012345678901234567890121 2
1
2345678901234567890123456789012123456789012345678901234567890121 2
1
2345678901234567890123456789012123456789012345678901234567890121 2
1
2345678901234567890123456789012123456789012345678901234567890121 2
1
2345678901234567890123456789012123456789012345678901234567890121 2
1
2345678901234567890123456789012123456789012345678901234567890121 2
1
2345678901234567890123456789012123456789012345678901234567890121 2
1
2345678901234567890123456789012123456789012345678901234567890121 2
1
2345678901234567890123456789012123456789012345678901234567890121 2
1
2345678901234567890123456789012123456789012345678901234567890121 2
1
2345678901234567890123456789012123456789012345678901234567890121 2
1
2345678901234567890123456789012123456789012345678901234567890121 2
1
2345678901234567890123456789012123456789012345678901234567890121 2
1
2345678901234567890123456789012123456789012345678901234567890121 2
1
2345678901234567890123456789012123456789012345678901234567890121 2
1
2345678901234567890123456789012123456789012345678901234567890121 2
1
2345678901234567890123456789012123456789012345678901234567890121 2
1
2345678901234567890123456789012123456789012345678901234567890121 2
1
2345678901234567890123456789012123456789012345678901234567890121 2
1
2345678901234567890123456789012123456789012345678901234567890121 2
1
2345678901234567890123456789012123456789012345678901234567890121 2
1
2345678901234567890123456789012123456789012345678901234567890121 2
1
2345678901234567890123456789012123456789012345678901234567890121 2
1
2345678901234567890123456789012123456789012345678901234567890121 2
1
2345678901234567890123456789012123456789012345678901234567890121 2
1
2345678901234567890123456789012123456789012345678901234567890121 2
1
2345678901234567890123456789012123456789012345678901234567890121 2
1
2345678901234567890123456789012123456789012345678901234567890121 2
1
2345678901234567890123456789012123456789012345678901234567890121 2
1
2345678901234567890123456789012123456789012345678901234567890121 2
1
2345678901234567890123456789012123456789012345678901234567890121 2
1
2345678901234567890123456789012123456789012345678901234567890121 2
1
2345678901234567890123456789012123456789012345678901234567890121 2
1
2345678901234567890123456789012123456789012345678901234567890121 2
1
2345678901234567890123456789012123456789012345678901234567890121 2
1
2345678901234567890123456789012123456789012345678901234567890121 2
1
2345678901234567890123456789012123456789012345678901234567890121 2
1
2345678901234567890123456789012123456789012345678901234567890121 2
1
2345678901234567890123456789012123456789012345678901234567890121 2
1
2345678901234567890123456789012123456789012345678901234567890121 2
1
2345678901234567890123456789012123456789012345678901234567890121 2
1
2345678901234567890123456789012123456789012345678901234567890121 2
1
2345678901234567890123456789012123456789012345678901234567890121 2
1
2345678901234567890123456789012123456789012345678901234567890121 2
1
2345678901234567890123456789012123456789012345678901234567890121 2
1
2345678901234567890123456789012123456789012345678901234567890121 2
1
2345678901234567890123456789012123456789012345678901234567890121 2
1
2345678901234567890123456789012123456789012345678901234567890121 2
1
2345678901234567890123456789012123456789012345678901234567890121 2
1
2345678901234567890123456789012123456789012345678901234567890121 2
1
2345678901234567890123456789012123456789012345678901234567890121 2
1
2345678901234567890123456789012123456789012345678901234567890121 2
1
2345678901234567890123456789012123456789012345678901234567890121 2
1
2345678901234567890123456789012123456789012345678901234567890121 2
1
2345678901234567890123456789012123456789012345678901234567890121 2
1
12345678901234567890123456789012123456789012345678901234567890121 2
123456789012345678901234567890121234567890123456789012345678901212
www.petersons.com
190
LSAT Success
QUICK-SCORE ANSWERS
QUICK-SCORE ANSWERS
Section 1
1. C
2. E
3. D
4. D
5. C
6. E
7. D
8. E
9. A
10. B
11. D
12. B
13. E
14. C
15. B
16. C
17. A
18. B
19. A
20. B
21. A
22. D
23. D
24. A
LSAT Success
Section 2
1. D
2. B
3. C
4. B
5. C
6. C
7. C
8. C
9. A
10. A
11. E
12. E
13. B
14. C
15. A
16. C
17. C
18. A
19. A
20. C
21. B
22. C
23. D
24. B
Section 3
1. A
2. D
3. A
4. B
5. A
6. A
7. A
8. C
9. D
10. A
11. E
12. A
13. C
14. C
15. C
16. E
17. E
18. C
19. A
20. B
21. B
22. E
23. B
24. E
Section 4
1. C
2. B
3. D
4. B
5. A
6. C
7. D
8. E
9. E
10. D
11. B
12. D
13. C
14. A
15. C
16. E
17. B
18. D
19. C
20. E
21. D
22. D
23. C
24. A
25. B
26. E
27. D
28. B
191
Section 5
1. A
2. C
3. B
4. C
5. C
6. B
7. C
8. A
9. E
10. D
11. D
12. C
13. C
14. C
15. E
16. C
17. E
18. A
19. A
20. D
21. C
22. B
23. C
24. C
www.petersons.com
EXPLANATORY ANSWERS
EXPLANATORY ANSWERS
Questions 712
Houses
Smaller
SECTION 1
Questions 16
Statehood
T
T's Children
Brother
Parents
Larger
A, F, K, O, T, V, W
Earlier
Later
Questions 1318
RidersA, B, C, D, E, F, G
Busses1, 2, 3
Bus
1 if B then no E or G
2 if D then no G
3 C always
4 when A and F take the same bus
13. The correct answer is (E). B, D, G
14. The correct answer is (C). 3 only
15. The correct answer is (B). 4
16. The correct answer is (C). 3 only
17. The correct answer is (A). 0
18. The correct answer is (B). A, D, E, G
www.petersons.com
192
LSAT Success
PRACTICE TEST 1
Questions 1924
W
B
D
E
B
E
5
7
2
3
A
C
C
C
5
7
2
3
D
E
B
IssueExtension question/weakening
evidence
SECTION 2
IssueExtension question/conclusion
IssueExtension question/conclusion
2. The correct answer is (B). Even though
all wrongs cannot be compensated for,
some wrongs can be.
PointMoney cannot compensate for
historic wrongs.
IssueExtension question/assumption
LSAT Success
193
www.petersons.com
EXPLANATORY ANSWERS
10. The correct answer is (A). a country in
which the majority of people enjoy a higher
standard of living than that of the American
people
PointAll Americans do not enjoy a high
standard of living.
IssueExtension question/weakening
evidence
17. The correct answer is (C). The significance of art is consistent over time.
PointSimilar results connote similar
causes, regardless of context.
IssueExtension question/assumption
18. The correct answer is (A). All the
enemies of the republic are dead.
PointAny living critic proves that Mao did
not have personal enemies killed.
IssueExtension question/assumption
19. The correct answer is (A). scientists are
too concerned about failure
PointRisking failure is no longer tolerated
in drug development.
IssueExtension question/conclusion
IssueExtension question/conclusion
12. The correct answer is (E). presenting a
hypothetical case
PointCensorship is appropriate in some
circumstances.
IssueDescription question/tactic
IssueExtension question/conclusion
IssueExtension question/conclusion
www.petersons.com
194
LSAT Success
PRACTICE TEST 1
Questions 712
Seating
H, I, J, K, L, M
Pres
Pres/VP= L or J
Secretary
Treasurer
VP
Trustee 1 H and/or I
Trustee 2
SECTION 3
opposite
T-1
not next to
J
or
K
T-2
Questions 16
North
Location
A, B, C, D, E, G
(N of C)
(W of D) A
Questions 1318
B
C (E of B)
Trucks - K, M, W
O - central
Vans - C, D, F
G (E of B)
Larger
(W of A)E
Small
D
K
equals
W more than C
LSAT Success
195
www.petersons.com
EXPLANATORY ANSWERS
Questions 1924
Smallest
Largest
Groups 1- 6
5
2 4
6 3 1 - not stone
SECTION 4
In the following answer guide, the credited
responses appear in bold type and the guide that
directs you to the place in the passage that
accounts for the credited response appears
within the answer-choice context. The first
reference is to the paragraph number in the
passage that accounts for the credited response
and the second number refers to the relevant
sentence in the paragraph. The reference will
appear as 2/4 for example. This means paragraph
2 and sentence 4 within paragraph 2.
11. The correct answer is (B). contrast judgments of right and wrong with other types of
judgmentsParagraph 4/Sentence 5
12. The correct answer is (D). means used to
produce resultsParagraph 3/Sentence 1
and Paragraph 2/Sentence 2
13. The correct answer is (C). explains some
judgments in law and moralsParagraph
1/Sentence 2
14. The correct answer is (A). somewhat
criticalParagraph 1/Sentence 4
3. The correct answer is (D). the description of the idealized competitive economy
Paragraph 4/Sentence 2
4. The correct answer is (B). concerned
with improving the operation of society
Paragraph 1/Sentence 4
5. The correct answer is (A). a recitation of
methods of approaching social problems
Most inclusive of the five answer choices
www.petersons.com
196
LSAT Success
PRACTICE TEST 1
SECTION 5
IssueExtension question/strengthening
evidence
4. The correct answer is (C). The sky is not
cloudy, and rain is not falling.
PointBob is not in the slicker he wears
when it is cloudy and rainy.
IssueExtension question/conclusion
LSAT Success
197
www.petersons.com
EXPLANATORY ANSWERS
7. The correct answer is (C). Some oceanliner passengers are in the care of a
dangerous person.
IssueExtension question/conclusion
10. The correct answer is (D). In the future,
more older students will enter college than
ever before.
PointCollege enrollment will decline
when the number of 18-year-olds
declines.
IssueExtension question/weakening
evidence
www.petersons.com
198
LSAT Success
PRACTICE TEST 1
20. The correct answer is (D). Nature cannot
be changed.
PointA snake will bite a hand that fed it.
IssueExtension question/conclusion
LSAT Success
199
www.petersons.com
EXPLANATORY ANSWERS
www.petersons.com
200
LSAT Success
Practice Test 2
SECTION 1
TIME35
(A)
(B)
(C)
(D)
(E)
(A)
(B)
(C)
(D)
(E)
(A)
(B)
(C)
(D)
(E)
(A)
(B)
(C)
(D)
(E)
C finishes ahead of D.
D finishes ahead of F.
F finishes ahead of D.
D finishes behind G.
G finishes behind F.
C finishes first.
G finishes first.
F finishes third.
D finishes fourth.
E finishes fourth.
E
F
F
E
F
1
2
3
4
5
5. Which of the following additional conditions makes it certain that F finishes second?
(A)
(B)
(C)
(D)
(E)
G finishes first.
C finishes first.
E finishes ahead of F.
F finishes ahead of E.
F finishes behind D.
Questions 16
(A)
(B)
(C)
(D)
(E)
24 QUESTIONS
MINUTES
second.
second.
third.
ahead of F.
ahead of D.
201
D finishes ahead of G.
H finishes ahead of E.
E finishes third.
F finishes fourth.
H finishes fifth.
PRACTICE TEST 2
10. If the Vermont president is younger than
the Maine president, the Wyoming president
is older than the Maine president, and the
Ohio president is the youngest, which of
the following is the second-oldest president?
Questions 712
The state presidents of the Half Century Club are
comparing ages at the Clubs annual meeting.
The Kansas president is older than the
Wyoming president.
(A)
(B)
(C)
(D)
(E)
Vermont
Wyoming
Maine
Kansas
Ohio
www.petersons.com
202
LSAT Success
SECTION 1
16. Train Q, a northbound express; train R, a
round-trip express coming from the south;
train S, a northbound metroliner; and trains
T and U, both southbound locals, are
arriving at the station, though not necessarily in that order.
Questions 1318
There are four parallel train tracks at a railroad
station, numbered 1 through 4 from left to right.
Tracks 1 and 2 are northbound, tracks 3 and 4
are southbound. A train coming from the north
will arrive on a southbound track. A train coming
from the south will arrive on a northbound track.
I. P is an express.
II. P and R are metroliners.
III. Q and R are locals.
(A)
(B)
(C)
(D)
(E)
(A)
(B)
(C)
(D)
(E)
I only
II only
III only
I and II only
I, II, and III
2,3,4,1
1,2,3,4
3,2,1,4
1,3,2,4
4,1,3,2
3,0
3,2
4,2
2,1
4,0
1
2
3
4
5
LSAT Success
203
www.petersons.com
PRACTICE TEST 2
21. If Walt brings the food, Rick CANNOT
Questions 1924
Five friends, Carol, Ed, Jenny, Rick, and Walt, go
to the beach. Each person either brings something (food, blankets, or umbrella) or does
something for the trip (drives or pays the tolls
along the route).
(A)
(B)
(C)
(D)
(E)
(A)
(B)
(C)
(D)
(E)
drive
be Jennys brother
bring the umbrella
be Carols brother
pay the tolls
drive
pay tolls
bring the blankets
bring the umbrella
be someones brother
(A)
(B)
(C)
(D)
(E)
Carol and Ed
Carol and Rick
Jenny and Walt
Carol and Walt
Jenny and Ed
www.petersons.com
204
LSAT Success
SECTION 2
SECTION 2
TIME35
MINUTES
24 QUESTIONS
(A) Arthritis specialists provide authoritative information on effective medication for arthritis.
(B) Arthritis specialists use Arthrelief because
it contains anti-inflammation medicine.
(C) A medicine containing ingredients prescribed for arthritis will be effective.
(D) Arthrelief only combats arthritic pain.
(E) Arthritis specialists prescribe Arthrelief
most.
Which of the following conclusions can be reasonably drawn from the information above?
LSAT Success
205
www.petersons.com
PRACTICE TEST 2
5. Japanese workers exercise each day in their
workplaces. American employers do not
require daily exercise, and, as a result,
American workers are more overweight and
much less fit and suffer more sickness and
injuries than their Japanese counterparts.
American workers will only become as
productive as the Japanese if they are
required to exercise on a daily basis.
(A)
(B)
(C)
(D)
(E)
www.petersons.com
I, II, III, IV
I, III, IV, II
II, III, IV, I
III, IV, I, II
IV, II, III, I
206
LSAT Success
SECTION 2
9. Ninety percent of the students taking as few
as two Alert tablets daily got better grades in
school. Improve your grades, get Alert today!
LSAT Success
207
www.petersons.com
PRACTICE TEST 2
16. Which of the following is assumed by the
above argument?
Questions 1617
Every time a business grants financial credit to an
individual, the business assumes the risk of the
individual not being able to make all the agreedupon payments. Credit bureaus assist businesses
in their efforts to evaluate the risks involved with
the extension of credit to individual purchasers.
The financial history of individuals is maintained
and reported on by credit bureaus. Credit
bureaus assist debtors as well as creditors by
preventing them from assuming greater debt
resulting from the work of credit bureaus, which
holds losses and prices down and, thus, benefits
consumers generally. The few concerns for
individual privacy that have been raised about
credit bureaus hardly offset their financial value
to business and consumer alike.
www.petersons.com
208
LSAT Success
SECTION 2
19. The average salary of a college graduate is
only 22 percent greater than that of a high
school graduate. In 1969, the difference was
55 percent. In addition, college graduates
salaries have not kept up with inflation. For
most, the rewards of a college education
will not justify the cost of tuition and lost
income while getting a degree.
Questions 2122
There is an inherent fallacy in the reasoning of
Shea, who suggests that all of his readers ought
to attend the retrospective of Beatles music
offered by the Springfield Pops because the
Beatles have had as great an influence on the
musical development of our day as Beethoven
had on his. Stalin had great influence on the
political development of his time, but no one
suggests that people should rush to Moscow to
pay homage at his tomb.
20. Genetic engineering places the natureversus-nurture argument in stark relief. Not
only will physical qualities (nature) of
individuals be altered by manufacturing
processes, but intellectual, emotional, and
spiritual qualities will be modified as well.
Those who argue that the altering of human
qualities violates the laws of nature ignore
the reality that people are already the result
of engineering in the form of their manufactured education, socialization, and environment (nurture).
Which of the following, if true, supports the
above argument?
LSAT Success
209
www.petersons.com
PRACTICE TEST 2
24. A recent survey has found that the number
of high school students that attend a house
of worship regularly has increased 60
percent in the past decade. This increase in
religious exposure appears to have significantly reduced cheating on tests and
improved class participation.
Which of the following, if true, most
weakens the inference made above?
(A) Religious leaders frequently speak out
for academic honesty.
(B) Not all students responded to the
survey.
(C) Recently the high school changed
from proctored exams to an honor
system.
(D) Social reasons account for the attendance of most students at services.
(E) Cheating is not considered to be a
major problem by most teachers.
www.petersons.com
210
LSAT Success
SECTION 3
SECTION 3
TIME35
(A)
(B)
(C)
(D)
(E)
(A)
(B)
(C)
(D)
(E)
(A)
(B)
(C)
(D)
(E)
M only
L only
Q or M
M or L
Q or L
LSAT Success
Q and N
M and Q
Q and L
L and P
Q and M
N
N
N
N
N
Questions 712
L, M, Q, O, P,
Q, M, L, O, P,
O, M, L, Q, P,
L, M, Q, P, O,
L, M, P, O, Q,
M, Q, O, P, L,
M, L, O, Q, P,
L, Q, M, P, O,
Q, M, O, L, P,
L, M, Q, N, P,
Questions 16
(A)
(B)
(C)
(D)
(E)
24 QUESTIONS
MINUTES
(A)
(B)
(C)
(D)
(E)
211
S, T; P, R; M, O; Q, N; L, U
S, T; P, R; M, N; Q, O; L, U
O, U; M, N; Q, L; P, R; S, T
P, R; M, U; Q, L; N, O; S, T
L, U; M, N; Q, O; P, R; S, T
www.petersons.com
PRACTICE TEST 2
8. Which of the following could be in the
pairs that graduated in 1971, 1973, and
1975, respectively?
(A)
(B)
(C)
(D)
(E)
Questions 1318
On any day their schedules permit, Adam, Beth,
Cary, Dana, and Edith each set up a sales table at
a flea market.
O, U, N
O, L, U
L, N, S
S, N, P
L, O, U
L graduated in 1972.
N graduated in 1974.
P graduated two years before N.
R graduated before 1974.
S graduated the year before O.
(A)
(B)
(C)
(D)
(E)
N graduated in 1974.
N graduated in 1972.
L graduated in 1974.
R graduated in 1972.
T graduated in 1971.
(A)
(B)
(C)
(D)
(E)
1971
1971
1971
1972
1972
and
and
and
and
and
(A)
(B)
(C)
(D)
(E)
P graduated in 1971.
O graduated in 1972.
Q graduated in 1972.
N graduated in 1973.
S graduated in 1975.
www.petersons.com
one
two
three
four
five
1972
1974
1975
1974
1975
Adam only
Beth only
Cary only
Adam and Beth only
Beth and Cary only
(A)
(B)
(C)
(D)
(E)
212
LSAT Success
SECTION 3
21. If T and V both are assigned to the same
two dentists, S must work for
(A)
(B)
(C)
(D)
(E)
Beth only
Cary only
Edith only
Cary and Edith only
Beth and Edith only
(A)
(B)
(C)
(D)
(E)
Zero
One
Two
Three
Four
A only
B only
C only
both A and B
both A and C
Questions 1924
(A)
(B)
(C)
(D)
(E)
S is assigned to C.
V is assigned to two dentists.
U is assigned to only one dentist.
T is assigned to C.
U is assigned to A.
both A and B
both A and C
either A or B
either A or C
either B or C
both A and B
both A and C
either A or B
either A or C
either B or C
LSAT Success
213
www.petersons.com
PRACTICE TEST 2
SECTION 4
TIME35 MINUTES
45
50
Line
10
15
20
25
30
35
40
www.petersons.com
55
60
65
70
75
80
85
214
28 QUESTIONS
LSAT Success
SECTION 4
1. The primary purpose of the passage is to
LSAT Success
Line
10
215
www.petersons.com
PRACTICE TEST 2
15
20
25
30
35
40
45
50
55
60
65
www.petersons.com
70
75
80
216
LSAT Success
SECTION 4
Line
It
It
It
It
It
can
can
can
can
can
10
15
20
30
35
40
LSAT Success
45
50
217
www.petersons.com
PRACTICE TEST 2
55
60
65
70
a sociologist
a genealogist
a heraldrist
an anthropologist
a legal historian
wealth
occupation
title
court performance
power
www.petersons.com
218
LSAT Success
SECTION 4
40
21. The passage suggests that all of the following would be advantages of using court
records and performance as a means of
social status determination EXCEPT
(A) court records can be reconstructed
over long periods of time
(B) court appearances change with status
within the society
(C) court records are generally preserved
in most jurisdictions
(D) court appearances can be treated
collectively to reflect status
(E) court procedures remain the same
over long periods of time
Line
10
15
20
25
30
35
45
50
55
LSAT Success
60
65
70
75
219
www.petersons.com
PRACTICE TEST 2
23. According to the passage, the consumer can
determine the quality of experience goods
by which of the following?
(A)
(B)
(C)
(D)
(E)
advertising
inspection
consumption
policing
reputation
www.petersons.com
220
LSAT Success
SECTION 5
SECTION 5
TIME35
MINUTES
24 QUESTIONS
LSAT Success
221
www.petersons.com
PRACTICE TEST 2
Questions 56
artist who was not actively hostile to his environment and, thus, an indifferent patriot. From
Dante to Tolstoy and from Shakespeare to Mark
Twain, the story is ever the same. Names suggest
themselves instantly: Goethe, Shelley, Byron,
Balzac, Cervantes, Swift, Dostoevsky, Carlyle,
Moliere, and Pope were each a bitter critic of his
time and nation.
Questions 78
It is almost as safe to assume that an artist of any
dignity is against his country, i.e., against the
environment in which God hath placed him, as it
is to assume that his country is against the artist.
He differs from the rest of us mainly because he
reacts sharply and in an uncommon manner to
phenomena that leave the rest of us unmoved,
or, at most, merely annoy us vaguely. Therefore,
he takes to artistic endeavor, which is at once a
criticism of life and an attempt to escape from
life.
The more the facts are studied, the more
they bear out these generalizations. In those
fields of art, at all events, which concern
themselves with ideas as well as with sensations,
it is almost impossible to find any trace of an
www.petersons.com
222
LSAT Success
SECTION 5
11. The author assumes which of the following
to be fact rather than opinion?
Questions 1112
The proposal to divert one third of the flow of
the Delaware River to supply New York City
with water ought to be a matter of great concern
to the people who live in the Delaware Valley.
The interests of the people of the Delaware
Valley are being put aside so that growth can
continue in an already overdeveloped area. Fresh
water is a natural resource in the same sense that
oil and coal are natural resources. Do Texas and
Alaska give away their oil? Does West Virginia
give away its coal? Why should Pennsylvania and
New Jersey supply New York or any other place
with fresh water? If the growth of New York is
capped by limited fresh water, so much the
better.
LSAT Success
223
www.petersons.com
PRACTICE TEST 2
14. Baxter defends paternal authority and the
preservation of the family in her most
recent work. But other aspects of her
thinking more convincingly demonstrate
that she cannot be considered a feminist.
For example, she fails to appreciate that the
full realization of a womans capacities
depends on her securing the same political,
economic, and civil rights as those afforded
me.
Question 1718
United States treaty negotiations with Japan
about trade involve the basic question, Can the
Japanese be trusted? But treaties are based on
self-interest rather than on trust. There would be
no need to have treaties if countries trusted one
another. A treaty is an alternative to trust; one
that formally recognizes that each country finds
an advantage in the agreement.
www.petersons.com
224
LSAT Success
SECTION 5
18. Which of the following is NOT supported
by the authors argument above?
LSAT Success
225
www.petersons.com
PRACTICE TEST 2
24. The argument above is most similar to
which of the following?
Questions 2324
The drug Thalidomide caused unforeseen birth
defects in thousands of babies; therefore,
thorough testing of the effects of all new drugs
should be required before release to the public.
www.petersons.com
226
LSAT Success
123456789012345678901234567890121234567890123456789012345678901212
2345678901234567890123456789012123456789012345678901234567890121 2
1
2345678901234567890123456789012123456789012345678901234567890121 2
1
2345678901234567890123456789012123456789012345678901234567890121 2
1
2345678901234567890123456789012123456789012345678901234567890121 2
1
2345678901234567890123456789012123456789012345678901234567890121 2
1
2345678901234567890123456789012123456789012345678901234567890121 2
1
2345678901234567890123456789012123456789012345678901234567890121 2
1
2345678901234567890123456789012123456789012345678901234567890121 2
1
2345678901234567890123456789012123456789012345678901234567890121 2
1
2345678901234567890123456789012123456789012345678901234567890121 2
1
2345678901234567890123456789012123456789012345678901234567890121 2
1
2345678901234567890123456789012123456789012345678901234567890121 2
1
2345678901234567890123456789012123456789012345678901234567890121 2
1
2345678901234567890123456789012123456789012345678901234567890121 2
1
2345678901234567890123456789012123456789012345678901234567890121 2
1
2345678901234567890123456789012123456789012345678901234567890121 2
1
2345678901234567890123456789012123456789012345678901234567890121 2
1
2345678901234567890123456789012123456789012345678901234567890121 2
1
2345678901234567890123456789012123456789012345678901234567890121 2
1
2345678901234567890123456789012123456789012345678901234567890121 2
1
2345678901234567890123456789012123456789012345678901234567890121 2
1
2345678901234567890123456789012123456789012345678901234567890121 2
1
2345678901234567890123456789012123456789012345678901234567890121 2
1
2345678901234567890123456789012123456789012345678901234567890121 2
1
2345678901234567890123456789012123456789012345678901234567890121 2
1
2345678901234567890123456789012123456789012345678901234567890121 2
1
2345678901234567890123456789012123456789012345678901234567890121 2
1
2345678901234567890123456789012123456789012345678901234567890121 2
1
2345678901234567890123456789012123456789012345678901234567890121 2
1
2345678901234567890123456789012123456789012345678901234567890121 2
1
2345678901234567890123456789012123456789012345678901234567890121 2
1
2345678901234567890123456789012123456789012345678901234567890121 2
1
2345678901234567890123456789012123456789012345678901234567890121 2
1
2345678901234567890123456789012123456789012345678901234567890121 2
1
2345678901234567890123456789012123456789012345678901234567890121 2
1
2345678901234567890123456789012123456789012345678901234567890121 2
1
2345678901234567890123456789012123456789012345678901234567890121 2
1
2345678901234567890123456789012123456789012345678901234567890121 2
1
12345678901234567890123456789012123456789012345678901234567890121 2
123456789012345678901234567890121234567890123456789012345678901212
LSAT Success
227
www.petersons.com
PRACTICE TEST 2
123456789012345678901234567890121234567890123456789012345678901212
2345678901234567890123456789012123456789012345678901234567890121 2
1
2345678901234567890123456789012123456789012345678901234567890121 2
1
2345678901234567890123456789012123456789012345678901234567890121 2
1
2345678901234567890123456789012123456789012345678901234567890121 2
1
2345678901234567890123456789012123456789012345678901234567890121 2
1
2345678901234567890123456789012123456789012345678901234567890121 2
1
2345678901234567890123456789012123456789012345678901234567890121 2
1
2345678901234567890123456789012123456789012345678901234567890121 2
1
2345678901234567890123456789012123456789012345678901234567890121 2
1
2345678901234567890123456789012123456789012345678901234567890121 2
1
2345678901234567890123456789012123456789012345678901234567890121 2
1
2345678901234567890123456789012123456789012345678901234567890121 2
1
2345678901234567890123456789012123456789012345678901234567890121 2
1
2345678901234567890123456789012123456789012345678901234567890121 2
1
2345678901234567890123456789012123456789012345678901234567890121 2
1
2345678901234567890123456789012123456789012345678901234567890121 2
1
2345678901234567890123456789012123456789012345678901234567890121 2
1
2345678901234567890123456789012123456789012345678901234567890121 2
1
2345678901234567890123456789012123456789012345678901234567890121 2
1
2345678901234567890123456789012123456789012345678901234567890121 2
1
2345678901234567890123456789012123456789012345678901234567890121 2
1
2345678901234567890123456789012123456789012345678901234567890121 2
1
2345678901234567890123456789012123456789012345678901234567890121 2
1
2345678901234567890123456789012123456789012345678901234567890121 2
1
2345678901234567890123456789012123456789012345678901234567890121 2
1
2345678901234567890123456789012123456789012345678901234567890121 2
1
2345678901234567890123456789012123456789012345678901234567890121 2
1
2345678901234567890123456789012123456789012345678901234567890121 2
1
2345678901234567890123456789012123456789012345678901234567890121 2
1
2345678901234567890123456789012123456789012345678901234567890121 2
1
2345678901234567890123456789012123456789012345678901234567890121 2
1
2345678901234567890123456789012123456789012345678901234567890121 2
1
2345678901234567890123456789012123456789012345678901234567890121 2
1
2345678901234567890123456789012123456789012345678901234567890121 2
1
2345678901234567890123456789012123456789012345678901234567890121 2
1
2345678901234567890123456789012123456789012345678901234567890121 2
1
2345678901234567890123456789012123456789012345678901234567890121 2
1
2345678901234567890123456789012123456789012345678901234567890121 2
1
2345678901234567890123456789012123456789012345678901234567890121 2
1
2345678901234567890123456789012123456789012345678901234567890121 2
1
2345678901234567890123456789012123456789012345678901234567890121 2
1
2345678901234567890123456789012123456789012345678901234567890121 2
1
2345678901234567890123456789012123456789012345678901234567890121 2
1
2345678901234567890123456789012123456789012345678901234567890121 2
1
2345678901234567890123456789012123456789012345678901234567890121 2
1
2345678901234567890123456789012123456789012345678901234567890121 2
1
2345678901234567890123456789012123456789012345678901234567890121 2
1
2345678901234567890123456789012123456789012345678901234567890121 2
1
2345678901234567890123456789012123456789012345678901234567890121 2
1
2345678901234567890123456789012123456789012345678901234567890121 2
1
2345678901234567890123456789012123456789012345678901234567890121 2
1
2345678901234567890123456789012123456789012345678901234567890121 2
1
2345678901234567890123456789012123456789012345678901234567890121 2
1
2345678901234567890123456789012123456789012345678901234567890121 2
1
2345678901234567890123456789012123456789012345678901234567890121 2
1
2345678901234567890123456789012123456789012345678901234567890121 2
1
2345678901234567890123456789012123456789012345678901234567890121 2
1
2345678901234567890123456789012123456789012345678901234567890121 2
1
2345678901234567890123456789012123456789012345678901234567890121 2
1
2345678901234567890123456789012123456789012345678901234567890121 2
1
2345678901234567890123456789012123456789012345678901234567890121 2
1
2345678901234567890123456789012123456789012345678901234567890121 2
1
2345678901234567890123456789012123456789012345678901234567890121 2
1
2345678901234567890123456789012123456789012345678901234567890121 2
1
2345678901234567890123456789012123456789012345678901234567890121 2
1
2345678901234567890123456789012123456789012345678901234567890121 2
1
2345678901234567890123456789012123456789012345678901234567890121 2
1
2345678901234567890123456789012123456789012345678901234567890121 2
1
2345678901234567890123456789012123456789012345678901234567890121 2
1
2345678901234567890123456789012123456789012345678901234567890121 2
1
2345678901234567890123456789012123456789012345678901234567890121 2
1
2345678901234567890123456789012123456789012345678901234567890121 2
1
2345678901234567890123456789012123456789012345678901234567890121 2
1
2345678901234567890123456789012123456789012345678901234567890121 2
1
2345678901234567890123456789012123456789012345678901234567890121 2
1
2345678901234567890123456789012123456789012345678901234567890121 2
1
2345678901234567890123456789012123456789012345678901234567890121 2
1
2345678901234567890123456789012123456789012345678901234567890121 2
1
2345678901234567890123456789012123456789012345678901234567890121 2
1
2345678901234567890123456789012123456789012345678901234567890121 2
1
2345678901234567890123456789012123456789012345678901234567890121 2
1
2345678901234567890123456789012123456789012345678901234567890121 2
1
2345678901234567890123456789012123456789012345678901234567890121 2
1
2345678901234567890123456789012123456789012345678901234567890121 2
1
2345678901234567890123456789012123456789012345678901234567890121 2
1
2345678901234567890123456789012123456789012345678901234567890121 2
1
2345678901234567890123456789012123456789012345678901234567890121 2
1
2345678901234567890123456789012123456789012345678901234567890121 2
1
12345678901234567890123456789012123456789012345678901234567890121 2
123456789012345678901234567890121234567890123456789012345678901212
www.petersons.com
228
LSAT Success
QUICK-SCORE ANSWERS
QUICK-SCORE ANSWERS
Section 1
1. E
2. A
3. D
4. C
5. C
6. D
7. E
8. E
9. D
10. B
11. C
12. D
13. D
14. E
15. B
16. C
17. E
18. E
19. B
20. E
21. C
22. C
23. A
24. B
LSAT Success
Section 2
1. A
2. C
3. A
4. A
5. D
6. D
7. E
8. D
9. A
10. E
11. E
12. B
13. E
14. E
15. C
16. C
17. D
18. B
19. C
20. D
21. A
22. A
23. A
24. D
Section 3
1. B
2. D
3. B
4. B
5. A
6. C
7. A
8. C
9. A
10. D
11. B
12. E
13. A
14. A
15. D
16. A
17. E
18. C
19. E
20. E
21. E
22. A
23. C
24. E
Section 4
1. C
2. D
3. A
4. D
5. A
6. C
7. C
8. D
9. C
10. E
11. B
12. B
13. D
14. C
15. D
16. B
17. D
18. A
19. B
20. E
21. E
22. A
23. C
24. E
25. D
26. A
27. A
28. C
229
Section 5
1. B
2. A
3. B
4. C
5. C
6. B
7. D
8. C
9. A
10. D
11. B
12. C
13. B
14. E
15. A
16. E
17. D
18. D
19. E
20. C
21. D
22. E
23. A
24. D
www.petersons.com
EXPLANATORY ANSWERS
EXPLANATORY ANSWERS
Questions 712
In the following answer guide, the credited responses appear in bold type and the visualization
that makes the credited response clear appears before the answers to each question set. Use the visualization to guide you in determining the credited answer.
Ages
A, F, I, K, M, O, T, U, V, W
Younger
SECTION 1
Older
W
O
Questions 16
Golf
7. The correct answer is (E). The Kansas president is younger than the Ohio president.
C, D, E, F, G
Last
First
G (1)
or
G (5)
E
C (1)
Questions 1318
4
S
Roundtrip tracks
www.petersons.com
230
LSAT Success
PRACTICE TEST 2
Questions 1924
Beach
C, E, J, R, W
Food, Blankets, Umbrella, Drives, Pays
Brother - Sister
First brings umbrella
Driver = male
Brother pays
When E brings food - W not drive
When C or J brings blankets - R drives
SECTION 2
LSAT Success
231
www.petersons.com
EXPLANATORY ANSWERS
17. The correct answer is (D). justified by the
economic value to business and society
IssueExtension question/strengthening
evidence
Questions 2122
Questions 1617
16. The correct answer is (C). Purchasers
attempt to secure more credit than they can
afford.
www.petersons.com
232
LSAT Success
PRACTICE TEST 2
24. The correct answer is (D). Social reasons
account for the attendance of most students
at services.
SECTION 3
Questions 1318
Table set-ups
Questions 16
Bids
A, B, C, D, E
B must E
C must D
E not with C
D not with A
A always sets up
L, M, N, O, P, Q
Highest
Questions 1924
Dentists - A, B, C
Hygienists - S, T, U, V
Questions 712
Graduation
L, M, N, O, P, Q, R, S, T, U
1971
1975
1
Q not with N
LSAT Success
233
www.petersons.com
EXPLANATORY ANSWERS
SECTION 4
2. The correct answer is (D). gun ownership by people associated with violence has
greatly increasedParagraph 2/Sentence 7
16. The correct answer is (B). the relationship between group status and frequency of
court appearanceParagraph 2/Sentence 2
21. The correct answer is (E). court procedures remain the same over long periods of
time. According to the passage, choices (A),
(B), (C), and (D) are all true. For (A), court
records can be constructed over long
periods of time, see 3/5; for (B), court
appearances change with status within the
society, see 3/1; for (C), court records are
generally preserved in most jurisdictions,
see 3/4; and for (D), court appearances can
be treated collectively to reflect status,
see 3/5.
www.petersons.com
234
LSAT Success
PRACTICE TEST 2
4. The correct answer is (C). Sales of a book
are not always indicative of its value.
PointAble criticism anticipates the market
reaction.
IssueExtension question/weakening
evidence
Questions 56
5. The correct answer is (C). the expression
of deep feeling
PointEmotional societies produce better
writing than technical societies.
IssueDescription question/rephrasing
Questions 78
28. The correct answer is (C). Credencegoods consumers require statutory protection.Paragraph 4/Sentence 2
SECTION 5
IssueDescription question/characterize
the number
IssueDescription question/tactic
LSAT Success
235
www.petersons.com
EXPLANATORY ANSWERS
Questions 1112
Questions 1718
20. The correct answer is (C). Only legislators who do not favor the bill were polled.
PointSome legislators support the bills
and some do not.
IssueExtension question/conclusion
www.petersons.com
IssueExtension question/conclusion
236
LSAT Success
PRACTICE TEST 2
Questions 2324
LSAT Success
237
www.petersons.com
Appendix
PAYING FOR LAW SCHOOL
What financial aid programs are available
at each of the schools to which I am
applying?
THINK AHEAD
The first step in putting together your financial
plan comes from thinking about the future: the
loss of your income while youre attending
school, your projected income after you graduate, the annual rate of inflation, additional
expenses you will incur as a student and after
you graduate, and any loss of income you may
experience later from unintentional periods of
unemployment, pregnancy, or disability. The
cornerstone of thinking ahead is following a
step-by-step process.
BE PREPARED
239
APPENDIX
Estimated Loan Repayment Schedule Monthly Payments for Every $1,000 Borrowed
Rate
5
years
10
years
15
years
20
years
25
years
5%
8%
9%
10%
12%
14%
$18.87
20.28
20.76
21.74
22.24
23.27
$10.61
12.13
12.67
13.77
14.35
15.53
$ 7.91
9.56
10.14
10.75
12.00
13.32
$ 6.60
8.36
9.00
9.65
11.01
12.44
$ 5.85
7.72
8.39
9.09
10.53
12.04
To make sure your educational debt is manageable, you should borrow an amount that requires
payments of between 8 and 15 percent of your
starting salary.
The approximate monthly installments for
repaying borrowed principal at 5, 810, 12, and
14 percent are indicated in the chart above.
Use this table to estimate your monthly
payments on a loan for any of the five repayment
periods (5, 10, 15, 20, and 25 years). The
amounts listed are the monthly payments for a
$1000 loan for each of the interest rates. To
estimate your monthly payment, choose the
closest interest rate and multiply the amount of
the payment listed by the total amount of your
loan and then divide by 1,000. For example, for a
total loan of $15,000 at 9 percent to be paid
back over ten years, multiply $12.67 times
15,000 (190,050) divided by 1,000. This yields
$190.05 per month.
If youre wondering just how much of a
loan payment you can afford monthly without
running into payment problems, consult the
chart on page 241.
Of course, the best way to manage your
debt is to borrow less. While cutting your
personal budget may be one option, you also
may wish to investigate asking your family for
help. Although the federal government considers
you independent, your parents and family may
still be willing and able to help pay for your
graduate education. If your family is not open to
just giving you money, they may be open to
making a low-interest (or deferred-interest) loan.
Family loans usually have more attractive interest
www.petersons.com
240
LSAT Success
TO
REPAY?
$4,000
$3,500
$3,000
$2,500
Likely
Payment
Problems
$2,000
$1,500
$1,000
$100
$200
$300
$400
$500
$600
PLASTIC MANIA
INTO
Any section on managing debt would be incomplete if it didnt mention the responsible use of
credit cards. Most law students hold one or more
credit cards, and many students find themselves
in financial difficulties because of them. Here are
two suggestions: use credit cards only for
convenience, never for extended credit; and, if
you have more than one credit card, keep only
the one that has the lowest finance charge and
the lowest limit.
ONE
LSAT Success
241
www.petersons.com
APPENDIX
gift aid programs provide the cost of tuition and
fees plus a stipend to cover living expenses.
Some are based exclusively on financial need,
some exclusively on academic merit, and some
on a combination of need and merit. As a rule,
grants are awarded to those with financial need,
although they may require the recipient to have
expertise in a certain field. Fellowships and
scholarships often connote selectivity based on
abilityfinancial need is usually not a factor.
Federal Support
The provision of free federal money in the form
of grants and scholarships for the training of
future lawyers has never ranked very high as a
national priority. This is especially noticeable
compared with what is available to prospective
doctors, teachers, or scientific researchers. The
only notable federal grant program for lawyers is
the newly enacted Thurgood Marshall Legal
Educational Opportunity Program to aid lowincome, minority, or disadvantaged students
seeking a law school education. The Marshall
program, to be administered by the Department
of Education, provides information, preparation,
and financial assistance to students so they may
gain access to and complete a law school
education. The U.S. Department of Education or
one of the law schools that you are interested in
should have more details about this new
program.
State Support
Many states offer grants for graduate study. States
grant approximately $64 million per year to
graduate students. To qualify for a particular
states aid you must be a resident of that state.
Residency is established in most states after you
have lived there for at least twelve consecutive
months prior to enrolling in school. Many states
provide funds for in-state students only; that is,
funds are not transferable out of state. Contact
your state scholarship office to determine what
aid it offers.
TYPES
OF
AID AVAILABLE
Institutional Aid
Educational institutions using their own funds
provide between $2 and $3 billion in graduate
assistance in the form of fellowships, tuition
waivers, and assistantships. In the field of law,
most scholarship aid comes from the institutions
themselves. Consult each schools catalog for
information about their aid programs.
www.petersons.com
242
LSAT Success
RESEARCHING GRANTS
FELLOWSHIPS
LSAT Success
AND
243
www.petersons.com
APPENDIX
OF
LOANS
Most needy law students borrow to finance their
law programs. There are basically two sources of
student loansthe federal government and
private loan programs. You should read and
understand the terms of these loan programs
before submitting your loan application.
University-Specific Information
on the Web
Universities are now in the process of creating
Web financial aid directories. Applications of
admission can be downloaded from the Web to
start the graduate process. After that, detailed
information can be obtained on financial aid
processes, forms, and deadlines. Universityspecific grant and scholarship information can
also be found, and more may be learned about
financing information by using the Web than by
an actual visit to the school. Questions can be
answered on line.
Federal Loans
Federal Stafford Loans. The Federal Stafford
Loan Program offers government-sponsored,
low-interest loans to students either through the
Department of Education or a private lender,
such as a bank, credit union, or savings and loan
association.
There are two components of the Federal
Stafford Loan program. Under the subsidized
component of the program, the federal government pays the interest accruing on the loan
while you are enrolled in law school on at least a
half-time basis. Under the unsubsidized component of the program, you pay the interest on the
loan from the day proceeds are issued. Eligibility
for the federal subsidy is based on demonstrated
financial need as determined by the financial aid
office from the information you provide on the
Free Application for Federal Student Aid (FAFSA).
A cosigner is not required, since the loan is not
based on creditworthiness.
Although Unsubsidized Federal Stafford
Loans may not be as desirable as Subsidized
Federal Stafford Loans from the consumers
perspective, they are a useful source of support
for those who may not qualify for the subsidized
loans or who need additional financial assistance.
Eligible borrowers may borrow up to $8500
per year through the Subsidized Stafford Loan
Program, up to a maximum of $65,500, including
undergraduate borrowing. In addition to loans
through the Subsidized Stafford Loan Program,
law students may borrow up to an additional
www.petersons.com
244
LSAT Success
LSAT Success
Supplemental Loans
Many lending institutions offer supplemental loan
programs and other financing plans, such as the
ones described below, to students seeking
assistance in meeting their expected contribution
toward educational expenses.
245
www.petersons.com
APPENDIX
If you are considering borrowing through a
supplemental loan program, you should carefully
consider the terms of the program and be sure to
read the fine print. Check with the program
sponsor for the most current terms that will be
applicable to the amounts you intend to borrow
for graduate study. Most supplemental loan
programs for graduate study offer unsubsidized,
credit-based loans. In general, a credit-ready
borrower is one who has a satisfactory credit
history or no credit history at all. A creditworthy
borrower generally must pass a credit test to be
eligible to borrow or act as a cosigner for the
loan funds.
Many supplemental loan programs have a
minimum annual loan limit and a maximum
annual loan limit. Some offer amounts equal to
the cost of attendance minus any other aid you
will receive for graduate study. If you are
planning to borrow for several years of graduate
study, consider whether there is a cumulative or
aggregate limit on the amount you may borrow.
Often this cumulative or aggregate limit will
include any amounts you borrowed and have not
repaid for undergraduate or previous graduate
study.
The combination of the annual interest rate,
loan fees, and the repayment terms you choose
will determine how much the amount is that you
will repay over time. Compare these features in
combination before you decide which loan
program to use. Some loans offer interest rates
that are adjusted monthly, some quarterly, some
annually. Some offer interest rates that are lower
during the in-school, grace, and deferment
periods, and then increase when you begin
repayment. Most programs include a loan
origination fee, which is usually deducted from
the principal amount you receive when the loan
is disbursed and must be repaid along with the
interest and other principal when you graduate,
withdraw from school, or drop below half-time
study. Sometimes the loan fees are reduced if
you borrow from a qualified cosigner. Some
programs allow you to defer interest and/or
principal payments while you are enrolled in law
school. Many programs allow you to capitalize
your interest payments; the interest due on your
loan is added to the outstanding balance of your
loan, so you dont have to repay immediately,
but this increases the amount you owe. Other
programs allow you to pay the interest as you
go, which will reduce the amount you later have
to repay.
www.petersons.com
How to Apply
All applicants for federal aid must complete the
Free Application for Federal Student Aid (FAFSA).
This application must be completed after
January 1 preceding enrollment in the fall. On
this form you report your income and asset
information for the preceding calendar year and
specify which schools will receive the data. Two
to four weeks later youll receive an acknowledgment on which you can make any corrections.
The schools youve designated will also receive
246
LSAT Success
Tax Issues
Since the passage of the Tax Reform Act of 1986,
grants, scholarships, and fellowships may be
considered taxable income. That portion of the
grant used for payment of tuition and courserequired fees, books, supplies, and equipment is
excludable from taxable income. Grant support
for living expenses is taxable. A good rule of
thumb for determining the tax liability for grants
and scholarships is to view anything that exceeds
the actual cost of tuition, required fees, books,
supplies related to courses, and required
equipment as taxable.
Application Deadlines
Application deadlines vary. Some schools require
you to apply for aid when applying for admission; others require that you be admitted before
applying for aid. Aid application instructions and
deadlines should be clearly stated in each
schools application material. The FAFSA must be
filed after January 1 of the year you are applying
for aid, but the Financial Aid PROFILE can be
completed earlier, in October or November.
LSAT Success
247
www.petersons.com
APPENDIX
A FINAL NOTE
www.petersons.com
248
LSAT Success